86
Milan Janji´ c Predavanja iz Elementarne teorije brojeva 2013 god. Prirodno-matematiˇ cki fakultet Univerzitet u Banjoj Luci

Elementarna teorija brojeva

Embed Size (px)

DESCRIPTION

Predavanja iz Elementarne teorije brojeva prof. dr Milana Janjica

Citation preview

Page 1: Elementarna teorija brojeva

Milan Janjic

Predavanja iz Elementarne teorijebrojeva 2013 god.

Prirodno-matematicki fakultetUniverzitet u Banjoj Luci

Page 2: Elementarna teorija brojeva

Algebra Algebra

0.1 Uvod

Bez muke nema nauke.

Narodna poslovica

Ovo su predavanja koja se drze 2012-2013. godine i nastala su modifikova-

njem i popravkom gresaka iz prethodne verzije predavanja.

Vec vise od 2000 godina, teorija brojeva odusevljava i inspirise i mate-

maticare profesionalci i matematicare amatere. Razvija se uz ucesce velikog

broja matematicara iz svih dijelovima svijeta.

Mnogi matematicari koji se bave ovom granom matematike, na celu sa jed-

nim od najcuvenijih, Gotfridom Hardijem, smatrali su da ova divna nauka nema

prakticnih primjena. Cak se smatralo da joj ta neprimjenjivost i daje posebnu

ljepotu. Medutim, razvoj savremenih tehnologija od ove je nauke napravio pri-

mjenljivu nauku u brojnim oblastima ljudskog djelovanja, kao sto su umjetnost,

teorija kodiranja, kriptografija i kompjuterske nauke.

1

Page 3: Elementarna teorija brojeva

1Prvo predavanje

1.1 Euklidov algoritam

Matematika je kraljica svih nauka,

a aritmetika je kraljica matematike.

Karl Fridrih Gaus

Na pocetku cemo dokazati jednu znacajnu osobinu cijelih brojeva na osnovu

koje cemo izvesti Osnovni stav aritmetike, jednu od vaznih teorema u mate-

matici. Kao i do sada, sa Z cemo oznacavati cijele brojeve. Dokaz se bazira na

poznatom principu minimuma za nenegativne cijele brojeve koji glasi: Svaki

neprazan podskup skupa nenegativnih cijelih brojeva ima minimalni element.

Propozicija 1.1 (Princip dijeljenja sa ostatkom)

Neka su a, b ∈ Z i b = 0. Postoje jedinstveni cijeli brojevi q i r za koje je

a = qb+ r, 0 6 r < |b|. (1.1)

Dokaz

Posmatrajmo skup A = {a−tb : t ∈ Z}. Jasno je da se, bez obzira na vrijednostia i b, moze izabrati t tako da bude a− tb ≥ 0. To znaci da u skupu A mora biti

nenegativnih brojeva. Prema principu minimuma, u skupu A postoji najmanji

nenegativan broj. Oznacimo taj broj sa r. Slijedi da postoji cio broj q za koji

je a = qb+ r. Tvrdimo da je 0 6 r < |b|. Ako to nije slucaj, onda bismo imali

Page 4: Elementarna teorija brojeva

GLAVA 1. PRVO PREDAVANJE 1.1. EUKLIDOV ALGORITAM

r = |b|+ r′, pri cemu je 0 6 r′ < r. Tada je r′ = a− qb− |b| = a− (q± 1)b ∈ A,

a to je nemoguce, prema izboru broja r. Time je postojanje broja r dokazano.

Dokazimo da je broj r je jedinstven. Pretpostavimo suprotno, da razliciti

brojevi r1, r2, gdje je r1 < r2, zadovoljavaju uslov (1.1). Na osnovu toga imamo

q1b+ r1 = q2b+ r2, tj. 0 = r2 − r1 = (q1 − q2)b. Slijedi da je r2 − r1 visekratnik

od b, a to je nemoguce, jer je 0 < r2 − r1 < |b|. Iz cinjenice da je r jedinstven i

iz a = q1b+ r = q2b+ r slijedi q1 = q2, pa je i q jedinstven.

Broj r iz jednakosti (1.1) se naziva ostatkom pri dijeljenju broja a brojem

b, dok se q naziva cijelim dijelom ili kolicnikom.

Jednakost (1.1) se cesto pise u obliku

a ≡ r mod b,

i kaze se da je a jednako r po modulu b. Napomenimo da je ovu oznaku uveo

Gaus.

Ako je r = 0, onda se kaze da je a djeljiv sa b ili da b dijeli a, i pise b|a.Dakle, b|a ako i samo ako postoji q ∈ Z takav da je a = bq. Djeljivost je svakako

najzanimljivija osobina cijelih brojeva. U sljedecoj propoziciji dokazujemo neke

od njenih osnovnih osobina.

Propozicija 1.2

Za cijele brojeve vrijede sljedece tvrdnje.

1. 1|n za svaki cio broj n.

2. Ako je m = 0, tada m|0 i m|m.

3. Ako m|n i m|q, tada m|un+ vq, za svaka dva cijela broja u i v.

4. Ako m|1, tada m = 1 ili m = −1.

5. Ako m|n i n|m, tada m = ±n.

6. Ako m|n i n|r, tada m|r.

Dokaz

Sve se tvrdnje jednostavno dokazuju, neposredno iz definicije. Mi cemo za ilu-

straciju dokazati trecu tvrdnju.

Iz m|n i m|q slijedi n = am, q = bm, za neke a i b. Za proizvoljne u i v

vrijedi un+ vq = aum+ bvm = (au+ bv)m, pa vrijedi m|(un+ vq).

Jednakost a = b · c nazivamo netrivijalnom faktorizacijom broja a ako

su i b i c razliciti od ±1.

3

Page 5: Elementarna teorija brojeva

1.1. EUKLIDOV ALGORITAM GLAVA 1. PRVO PREDAVANJE

Definicija 1.3

Cio broj p > 1 nazivamo prostim ako za svaka dva cijela broja a i b iz p|abslijedi p|a ili p|b.

Propozicija 1.4

Ako je p prost broj i p|a1 · a2 · · · an, tada postoji indeks i za koji p|ai.

Dokaz

Za n = 1 tvrdnja je ocigledna, a za n = 2 slijedi iz definicije. Ako je tvrdnja

tacna za n−1 i ako p|a1 ·a2 · · · an, tada p|an ili p|a1 ·a2 · · · an−1. Ako p|an onda

je teorema dokazana, a u suprotnom p|ai, za neki 1 6 i 6 n − 1, na osnovu

induktivne pretpostavke.

Izlozicemo sada jedan zanimljiv Ojlerov rezultat o djeljivosti. Brojevi oblika

22n

+ 1, gdje n ≥ 0, nazivaju se Fermaovim brojevima. Prvih pet Fermaovih

brojeva su 3, 5, 17, 257, 65537, i svi su prosti. Ferma je pogresno mislio da su svi

Fermaovi brojevi prosti. Medutim, Ojler je dokazao da je 225

+ 1 slozen broj.

Primjer 1.5

Vrijedi 641|(22

5

+ 1).

Rjesenje. Ovo je rjesenje zanimljivo jer se u njemu eksplicitno dijeljenje uopste

ne pojavljuje. Uocimo da vrijedi:

641 = 5 · 27 + 1 = 24 + 54.

Prema tome,

225

+ 1 = 232 + 1 = 24 · 228 + 1 = (641− 54) · 228 + 1

= 641 · 228 − (5 · 27)4 + 1 = 641 · 228 − (641− 1)4 + 1

= 641(228 − 6413 + 4 · 6412 − 6 · 641 + 4).

Primijenicemo teoremu o dijeljenju sa ostatkom za dokaz da se svaki pri-

rodan broj moze jednoznacno prikazati u brojnom sistemu sa proizvoljnom

osnovom.

Teorema 1.6

Neka je b > 1 prirodan broj. Tada se svaki prirodan broj n moze prikazati na

4

Page 6: Elementarna teorija brojeva

GLAVA 1. PRVO PREDAVANJE 1.1. EUKLIDOV ALGORITAM

jedinstven nacin u obliku:

n = akbk + ak−1b

k−1 + · · ·+ a0,

pri cemu su a0, a1, . . . , ak nenegativni cijeli brojevi manji od b.

Dokaz

Na osnovu principa dijeljenja sa ostatkom imamo n = q1b+a0, 0 6 a0 < b. Ako

podijelimo q1 sa b, dobijemo q1 = q2b+a1, 0 6 a1 < b. Slijedi n = q2b2+a1b+a0.

Ponavljamo ovaj postupak do trenutka kada bude qk < b. Uzmemo ak = qk i

dobijemo trazeno razlaganje. Jedinstvenost razlaganja slijedi iz jedinstvenosti

ostatka pri dijeljenju.

Pored standardne baze 10 najcesce se koristi binarni sistem zapisa brojeva,

u kome je osnova 2.

Primjer 1.7

Broj ima sljedeci zapis u bazi 7 : (456)7. Zapisati taj broj u bazi 5.

Rjesenje. U dekadskom zapisu broj ima oblik: 4 · 72 + 5 · 7 + 6 = 237. Iz niza

dijeljenja sa 5 dobijamo:

237 = 47 · 5 + 2, 47 = 9 · 5 + 2, 9 = 1 · 5 + 4, 1 = 0 · 5 + 1

Prema tome, 237 = (1422)5.

Ako se za osnovu uzme 2, onda prethodna teorema ima sljedeci oblik.

Posljedica 1.8

Za svaki prirodan broj n postoji jedinstven niz mk > mk−1 > · · · > m0 > 0 za

koji jen = 2mk + 2mk−1 + · · ·+ 2m0 .

Pomocu prethodnog prikaza moguce je odrediti optimalnu strategiju u sta-

roj japanskoj igri NIM.

Primjer 1.9

Igru NIM igraju dva igraca na sljedeci nacin. Na tri hrpe nalaze se stapici. Prvi

igrac sa bilo koje od hrpa uzima najmanje jedan stapic, zatim isto cini drugi

igrac, itd. Pobjednik je onaj koji uzme posljednji stapic. Naci najbolju mogucu

strategiju za pobjedu u ovoj igri.

5

Page 7: Elementarna teorija brojeva

1.1. EUKLIDOV ALGORITAM GLAVA 1. PRVO PREDAVANJE

Rjesenje. Neka je p, q, r broj stapica na prvoj, drugoj i trecoj hrpi respektivno.

Trojku (p, q, r) nazivamo pozicijom. Predstavimo p, q, r u binarnom sistemu

p = ek2k + ek−12

k−1 + · · ·+ e0,

q = fk2k + fk−12

k−1 + · · ·+ f0,

r = gk2k + gk−12

k−1 + · · ·+ g0.

Za poziciju kazemo da je nepovoljna ako su svi brojevi ei + fi + gi, i =

0, 1, . . . , k, parni. Ostale pozicije nazivamo povoljnim. Npr. pozicija (0, 0, 0) je

nepovoljna. U ostalim sljucajevima u kojima je ei+fi+gi paran moraju dva od

ova tri broja biti jedinice, a jedan nula. U tom slucaju stapica ima bar na dvije

hrpe. Prema tome, ako je igrac u nepovoljnoj poziciji, on nema pobjednickog

poteza.

Pokazacemo da igrac koji se nalazi u nepovoljnoj poziciji, bilo kojim pote-

zom stavlja protivnika u povoljnu poziciju. Neka je npr. (p, q, r) nepovoljna pozi-

cija i neka igrac uzme nekoliko stapica sa jedne hrpe. Neka je npr. uzeo p−p′ ≥ 1

stapica sa prve hrpe. Nastaje pozicija (p′, q, r), gdje je p′ < p. Pokazimo da je

ona povoljna. Pomenuti zbirovi za ovu poziciju su oblika e′i+fi+gi, pri cemu je

bar jedan e′i manji od odgovarajuceg ei. Kada bi i ova pozicija bila nepovoljna

to bi znacilo da su svi brojevi ei + fi + gi, e′i + fi + gi parni. Razlika parnih

brojeva je paran broj, pa bi to znacilo da su svi ei − e′i parni. Medutim broj

ei − e′i moze biti samo 0 ili 1, pa bi moralo biti ei − e′i = 0, tj. ei = e′i, za svako

i, a to je nemoguce.

Dakle, strategija igre se sastoji u potezu koji treba odigrati u povoljnoj

poziciji. Ako se u povoljnoj poziciji svi stapici nalaze na jednoj hrpi, onda se

odmah dobija. Na kraju, pitanje je sta treba igrac raditi ako je u povoljnoj

poziciji, a stapici se nalaze na vise hrpa. Pokazacemo da se tada moze odigrati

tako da se protivnik stavi u nepovoljnu poziciju.

Pretpostavimo da je j najveci od indeksa 0, 1, 2, . . . , k za koji je ej +fj + gjneparan broj. Posto je pozicija povoljna, takav j postoji. Sigurno je bar jedan

od brojeva ej , fj , gj jednak 1. Neka je to ej . Dakle,

p = ek2k + · · ·+ 2j + · · ·+ e0.

Posmatrajmo sada broj

p′ = ek2k + · · ·+ 0 · 2j + xj−12

j−1 · · ·+ x0.

Jasno je p′ < p, kako god izabrali xj−1, . . . , x0.

Moguce je izabrati xi, i = 0, 1, . . . , j − 1, tako da fi + gi + xi bude paran

broj. Dakle, pozicija (p′, q, r) je nepovoljna i stvara se uzimanjem sa prve hrpe

p− p′ stapica.

Rijesicemo jos jedan zanimljiv zadatak, koji potice jos iz 17. vijeka.

6

Page 8: Elementarna teorija brojeva

GLAVA 1. PRVO PREDAVANJE 1.1. EUKLIDOV ALGORITAM

Primjer 1.10

Treba napraviti cetiri tega tako da je njihova ukupna tezina 40 kg, uz uslov da

se pomocu tih tegova i terazija moze izmjeriti svaka tezina od 1 do 40. Kolika

treba biti tezina svakog tega? Kako izmjeriti tezinu od 34 kg?

Rjesenje. Zanimljivo je da se rjesenje dobija uz pomoc zapisa brojeva u bazi 3.

Zapravo, taj zapis cemo malo modifikovati. Pokazimo da se svaki pozitivan cio

broj n moze napisati u obliku

n = ek3k + ek−13

k−1 + · · ·+ e13 + e0, ei ∈ {−1, 0, 1}, (i = 0, 1, . . . , k).

Zaista, neka je n = fk3k + fk−13

k−1 + · · · + f13 + f0 prikaz broja n u bazi 3.

Ako je svaki fi jednak 0 ili 1, onda se nema sta dokazivati. Neka je s najmanji

broj za koji je fs = 2. Zamijenjujuci fs sa 3− 1 dobijamo

n = fk3k + fk−13

k−1 + · · ·+ (fs+1 + 1)3s+1 − 3s + es−13s−1 + · · ·+ e0.

Znaci, es = −1.

Ako je fs+1 = 0, tada je es+1 = 1. Ako je fs+1 = 1, onda je es+1 = −1, dok

se fs+2 uvecava za 1. Na kraju, ako je fs+1 = 2, onda je es+1 = 0, dok se fs+2

ponovo uvecava za 1. Ponavljajuci ovaj postupak na kraju dobijamo zeljeni

prikaz. Prema tome, razlaganje postoji. Ako u tom razlaganju prebacimo na

lijevu stranu sabirke kod kojih je ei = −1, a izostavimo sabirke kod kojih je

e = 0, dobijamo

n+ 3i1 + 3i2 + · · ·+ 3it = 3j1 + 3j2 + · · ·+ 3jr .

Ova se jednakost moze interpretirati u terminima vaganja na terazijama. Ako

imamo tegove od 1, 3, 32, . . . , 3k kilograma i ako je x predmet neke tezine n,

koja nije veca od zbira tezina svih tegova, tj. n 6 1+3+32+ · · ·+3k = 3k+1−12 ,

tada na jedan tas vage stavimo predmet x i tegove tezine 3i1 , . . . , 3ir , a na drugi

tas tegove tezine 3j1 , 3j2 , . . . , 3jt . Vaga ce biti u ravnotezi ako predmet x ima

tezinu n.

Za rjesenje postavljenog problema trebaju nam tegovi od 1, 3, 9 i 27 kilo-

grama. Tezinu od 34 kg cemo izmjeriti na sljedeci nacin:

34 = 33 + 2 · 3 + 1 = 33 + 32 − 3 + 1.

Dakle, ako hocemo da provjerimo da li je neki predmet tezak 34 kilograma, na

jedan tas stavljamo taj predmet i teg od tri kilograma, a na drugi tegove od

27, 9 i 1 kilogram, pa ako je vaga u ravnotezi onda je predmet tezak 34 kg.

Definicija 1.11

Ako su a1, a2, . . . , an cijeli brojevi, koji nisu svi jednaki nuli, tada se najveci

cio broj koji dijeli sve ai naziva najvecim zajednickim djeliteljem tih brojeva

7

Page 9: Elementarna teorija brojeva

1.1. EUKLIDOV ALGORITAM GLAVA 1. PRVO PREDAVANJE

i oznacava se sa (a1, a2, . . . , an). Specijalno je (a, b) najveci zajednicki djelitelj

brojeva a i b.

Lako je dokazati da se najveci zajednicki djelitelj za vise brojeva moze biti

definisan rekurentno, tj. da vrijedi

(a1, a2, . . . , an) = ((a1, a2, . . . , an−1), an). (1.2)

Za cijele brojeve a1, a2, . . . , an, koji nisu svi jednaki nuli, definisemo naj-

manji zajednicki sadrzilac, kao najmanji pozitivan cio broj, koga dijele svi

ai, i = 1, 2, . . . , n. Za taj broj koristimo oznaku [a1, a2, . . . , an].

Izlozicemo sada Euklidov algoritam za odredivanje najveceg zajednickog

djelitelja dva cijela broja. Napomenimo da rijec algoritam znaci postupak za

rjesavanje nekog problema, koji u konacnom broju koraka dovodi do rjesenja

tog problema ili dokazuje da problem nema rjesenje.

Propozicija 1.12 (Euklidov algoritam)

Neka su a, b ∈ Z i b > 0. Ako su q i r jedinstveni cijeli brojevi takvi da je

a = qb+ r i 0 6 r < b, tada je (a, b) = (b, r).

Dokaz

Ako je r = 0 onda b|a, pa je (a, b) = b, sto znaci da je tvrdnja tacna u ovom

slucaju. Pretpostavimo zato da je r > 0. Kako (a, b)|a i (a, b)|b, prema propo-

ziciji 1.2, imamo (a, b)|r. Prema tome, (a, b) 6 (b, r). Isto tako, (b, r)|bq+ r, stoznaci (b, r)|a, pa (b, r) 6 (a, b). Slijedi da je (a, b) = (b, r).

Napomenimo da se ovdje stvarno radi o algoritmu, jer je ili r = 0, pa je u

tom slucaju (a, b) = b, ili je 0 < r < b, kada se postupak ponavlja tako da se

umjesto para a, b uzme par b, r. Kako je r < b, slijedi da ce se ponavljanjem,

u konacno mnogo koraka doci do ostatka koji je jednak nuli, kada se postupak

zavrsava.

Moguce je, dakle, po gornjem pravilu formirati konacan niz dijeljenja sa

ostatkom

a = q0b+ r1,

b = q1r1 + r2,

r1 = q2r2 + r3,...

rk−2 = qk−1rk−1 + rk,

rk−1 = qkrk + 0.

(1.3)

8

Page 10: Elementarna teorija brojeva

GLAVA 1. PRVO PREDAVANJE 1.1. EUKLIDOV ALGORITAM

Pri tome je

(a, b) = (b, r1) = (r1, r2) = . . . = (rk−1, rk) = (rk, 0) = rk.

Primjer 1.13

Odrediti (529, 1541, 1817).

Rjesenje. Prvo cemo odrediti (529, 1541). Imamo

1541 = 2 · 529 + 483,

529 = 1 · 483 + 46,

483 = 10 · 46 + 23,

46 = 2 · 43 + 0.

Dakle, (529, 1541) = 23. U skladu sa (1.2) vrijedi (529, 1541, 1817) = (23, 1817).

Kako je1817 = 23 · 79 + 0,

zakljucujemo da je (529, 1541, 1817) = 23.

U sljedecoj teoremi cemo pokazati da se najveci zajednicki djelitelj brojeva

a i b moze izraziti preko samih a i b.

Izraze oblika∑n

i=1 xiai nazivamo linearnim kombinacijama brojeva

a1, a2, . . . , an. Brojeve x1, x2, . . . , xn nazivamo koeficijentima linearne kom-

binacije.

Propozicija 1.14 (Bezuova formula)

Ako su a i b cijeli brojevi takvi da je bar jedan razlicit od 0, tada se (a, b) moze

dobiti kao linearna kombinacija brojeva a i b sa cijelim koeficijentima.

Dokaz

Izlozicemo dva dokaza ove teoreme.

1. Iz jednakosti (1.3) zakljucujemo da je r1 linearna kombinacija od a i b, r2je linearna kombinacija od b i r1, r3 je linearna kombinacija od r2 i r1, itd.

Broj rk je linearna kombinacija od rk−1 i rk−2. Odavde slijedi da

(a, b) = rk = s1rk−1 + t1rk−2 = s1(s2rk−2 + t2rk−3) + t1rk−2 =

= (s1s2 + t1)rk−2 + t2rk−3 = . . . = sa+ tb.

Napominjemo da je ovaj dokaz algoritamski, tj. pomocu njega mozemo

dobiti konkretne brojeve s i t za koje je (a, b) = sa + tb. Vidjecemo da je

to vazno u primjenama.

9

Page 11: Elementarna teorija brojeva

1.1. EUKLIDOV ALGORITAM GLAVA 1. PRVO PREDAVANJE

2. Za razliku od prvog, ovaj dokaz ce biti ,,egzistencijalan”, tj. bice dokazano

da trazena linearna kombinacije postoji, ali nece biti receno kako se ona

konkretno moze naci. Posmatrajmo skup {ua+ vb|u, v ∈ Z} svih linearnih

kombinacija od a i b sa cijelim koeficijentima. Neka je S skup svih pozitivnih

brojeva iz prethodnog skupa. Jasno je da je skup S neprazan. Na osnovu

principa minimuma, u skupu S postoji najmanji element d. Dokazimo da

je d = (a, b). Kako je d iz S, to postoje cijeli brojevi s i t takvi da je

d = sa+ tb. Imamo da d|a i d|b, jer ako npr. d ne bi dijelio a, onda bismo

imali a = qd+ r, (0 < r < d), sto znaci da je r = a− qd = a− q(sa+ tb) =

(1 − qs)a + qtb ∈ S, sto je nemoguce, jer je r < d. Prema tome, d dijeli

i a i b. Ako je c bilo koji djelitelj od a i b, tada, prema propoziciji 1.2,

c|sa+ tb = d, pa je c 6 d. Prema tome, d je najveci zajednicki djelitelj od

a i b.

Primjer 1.15

Izraziti (2012, 1941) kao linearnu kombinaciju od 2012 i 1941.

Rjesenje. Imamo2012 = 1 · 1941 + 71,

1941 = 27 · 71 + 24,

71 = 2 · 24 + 23,

24 = 1 · 23 + 1.

23 = 23 · 1 + 0.

Prema tome, (2012, 1941) = 1. Dalje imamo

1 = 24− 23 = 24− (71− 2 · 24) = −71 + 3 · 24= −71 + 3 · (1941− 27 · 71) = 3 · 1941− 82 · 71= 3 · 1941− 82 · (2012− 1941)

= 85 · 1941− 82 · 2012.

Vrijedi i opstija tvrdnja.

Propozicija 1.16

Ako su a1, a2, . . . , an cijeli brojevi od kojih je barem jedan razlicit od nule, tada

je (a1, a2, . . . , an) neka linearna kombinacija brojeva a1, a2, . . . , an.

10

Page 12: Elementarna teorija brojeva

GLAVA 1. PRVO PREDAVANJE 1.1. EUKLIDOV ALGORITAM

Dokaz

Dokaz vodimo indukcijom po n. Za n = 2 tvrdnja vrijedi na osnovu pret-

hodne propozicije. Pretpostavimo da je tvrdnja tacna za n − 1 > 2. Neka je

d1 = (a1, a2, . . . , an−1). Prema induktivnoj pretpostavci, postoje cijeli brojevi

y1, y2, . . . , yn−1, za koje je d1 = y1a1 + · · ·+ yn−1an−1.

Isto tako, postoje cijeli brojevi z1, z2, za koje je (d1, an) = z1d1 + z2an. Iz

jednakosti (1.2) slijedi

(a1, a2, . . . , an) = (d1, an) = z1(y1a1 + · · ·+ yn−1an−1) + z2an.

Definicija 1.17

Cijele brojeve a i b nazivamo relativno prostim ako je (a, b) = 1.

Iz prethodne propozicije neposredno slijedi sljedeca tvrdnja.

Propozicija 1.18

Cijeli brojevi a i b su relativno prosti ako i samo ako postoje cijeli brojevi x i

y takvi da

ax+ by = 1.

Propozicija 1.19

Ako (a, b) = 1 i a|bc, tada a|c.

Dokaz

Postoje cijeli brojevi u i v za koje je au + bv = 1. Mnozenjem sa c dobijamo

a(uc) + v(bc) = c. Kako a|bc, to a|c.

U sljedecoj tvrdnji dajemo dvije jednostavne karakterizacije prostih brojeva.

Propozicija 1.20

Sljedece tvrdnje su ekvivalentne.

1. Broj p je prost.

2. Za svaki cio broj a vrijedi p|a ili (a, p) = 1.

3. Broj p ima samo trivijalnu faktorizaciju.

11

Page 13: Elementarna teorija brojeva

1.1. EUKLIDOV ALGORITAM GLAVA 1. PRVO PREDAVANJE

Dokaz

Tvrdnju dokazujemo tako sto cemo dokazati niz implikacija 1.⇒ 2.⇒ 3.⇒ 1.

1.⇒ 2. Neka je p prost, a a proizvoljan cio broj. Ako p - a i ako je d = (p, a),

onda je p = d. Vrijedi p = dd′, pa p|d ili p|d′. Ako p|d, onda bi moralo biti p = d,

sto nije. Zbog toga p|d′, a to znaci da je p = d′, tj. d = 1.

2. ⇒ 3. Neka je p = ab, (a > 0, b > 0), faktorizacija broja p. Ako p|a,onda je p = a, pa je ta faktorizacija trivijalna. Ako p - a onda, prema b),

(p, a) = 1, tj. vrijedi xp + ya = 1, za neke cijele brojeve x i y. Mnozeci sa b

dobijamo xbp + yab = b, pa p|b, odnosno p = b. Prema tome, faktorizacija je,

opet, trivijalna.

3.⇒ 1. Neka p|ab. Ako p|a, tvrdnja je dokazana. Ako p - a i ako je d = (p, a),

tada je p = dd′, za neki pozitivan cio broj d′. Kako p ima samo trivijalnu

faktorizaciju slijedi d = 1, jer ako je d′ = 1, onda je d = p, pa bi bilo p|a. Znaci1 = xa+ yp, za neke cijele brojeve x i y slijedi b = xab+ ybp, pa p|b.

Kao posljedicu prethodne tvrdnje razmatracemo linearnu Diofantovu

jednacinu. Inace, Diofantovim se nazivaju jednacine u kojima se traze cje-

lobrojna rjesenja za nepoznate. Jednacina

ax+ by = c, (1.4)

pri cemu su a, b, c dati cijeli brojevi, a x i y nepoznati cijeli brojevi, se naziva

linearnom Diofantovom jednacinom.

Pretpostavimo da jednacina (12.6) ima rjesenje. Ako je d = (a, b), onda

imamo da d|ax + by, pa zakljucujemo da d|c. Dakle, d|c je potreban uslov da

jednacina (12.6) ima rjesenje. Dokazimo da je taj uslov i dovoljan. Na osnovu

propozicije 1.14 postoje cijeli brojevi u i v za koje vrijedi au + bv = d. Kako

d|c to znaci da je c = d · d′, za neki cio broj d′, pa je a(ud′)+ b(vd′) = c. Dakle,

x = ud′, y = vd′ je rjesenje jednacine (12.6), cime je tvrdnja dokazana.

Neka je (x0, y0) fiksirano, a (x, y) bilo koje rjesenje jednacine. Tada imamo

a(x− x0) = b(y0 − y). Dijeljenjem sa d dobijamo

a

d(x− x0) =

b

d(y0 − y).

Kako je(ad ,

bd

)= 1, na osnovu propozicije 1.19 zakljucujemo da je x−x0 = k b

d ,

za neki cio broj k. Uvrstavanjem dobijamo y0 − y = k ad . Sa druge strane, za

proizvoljan cio broj k lako je provjeriti da je

x = x0 + k · bd, y = y0 − k · a

d, (1.5)

rjesenje jednacine (12.6). Ovaj skup rjesenja nazivamo opstim rjesenjem.

Tako smo dokazali sljedecu propoziciju.

12

Page 14: Elementarna teorija brojeva

GLAVA 1. PRVO PREDAVANJE 1.1. EUKLIDOV ALGORITAM

Propozicija 1.21

Da bi jednacina ax + by = c imala rjesenje potrebno je i dovoljno da d|c, pricemu je d = (a, b). Ako je (x0, y0) jedno rjesenje, tada je opste rjesenje data sa

(1.5).

Primjedba 1.22

Primijetimo da je u prethodnom postupku opisano i kako se moze naci jedno

rjesenje (x0, y0). Naime, ako pomocu Euklidovog algoritma odredimo cijele bro-

jeve u i v za koje je au + bv = d, onda je x0 = ud′, y0 = vd′ trazeno rjesenje.

Pri tome je d′ = cd .

Primjer 1.23

Dokazati da se broj 713 ne moze napisati kao suma dva pozitivna broja, od

kojih je jedan djeljiv sa 22, a drugi sa 35.

Rjesenje. Treba dokazati da linearna Diofantova jednacina 22x + 35y = 713

nema pozitivnih rjesenja. Kako su 22 i 35 relativno prosti, to jednacina ima

rjesenje. Iz Euklidovog algoritma dobijamo: 1 = 8 · 22 − 5 · 35. Slijedi da je

x0 = 8 · 713 = 5704, y0 = −5 · 713 = −3565 partikularno rjesenje. Opste

rjesenje je

x = 5704 + k · 35, y = −3565− k · 22,

pri cemu je k bilo koji cio broj. Treba dokazati da ni za jedno k ne mogu i x, i y

istovremeno biti pozitivni. Problem se lako svodi na problem odredivanja cije-

log broja k za koji je −570435 < k < −3565

22 , a lako se vidi da takav broj ne postoji.

Sljedeci problem je specijalni slucaj tzv. problema razmjene novcanica,

koji je 1884. godine postavio Frobenijus, a glasi: Date su novcanice vrijednosti

n1, n2, . . . , nk, pri cemu je (n1, n2, . . . , nk) = 1. Odrediti najveci iznos koji se

ne moze isplatiti u ovim novcanicama (ovaj iznos se naziva Frobenijusovim

brojem).

Ako je neki od brojeva ni jednak 1, onda se jasno svaki iznos moze isplatiti.

Broj 3 je Frobenijusov broj za iznose od 2 i 5. Naime, jasno je da se iznos 3

ne moze isplatiti u datim novcanicama. Dalje, svaki paran iznos se, ocigledno

moze isplatiti. Ako je 2k+1, (k ≥ 2) neparan iznos, onda 2k+1 = 2(k−2)+5,

pa se i ovaj iznos moze isplatiti.

U sljedecem primjeru cemo odrediti Frobenijusove brojeve za dvije vrste

novcanica.

13

Page 15: Elementarna teorija brojeva

1.2. OSNOVNI STAV ARITMETIKE GLAVA 1. PRVO PREDAVANJE

Primjer 1.24

Za vrijednosti novcanica n1 i n2, Frobenijusov broj je n1 · n2 − n1 − n2.

Rjesenje. Iznos N moze biti isplacen ako postoje nenegativni cijeli brojevi x

i y za koje je xn1 + yn2 = N. Kako su n1 i n2 relativno prosti, na osnovu

propozicije 1.18, postoje cijeli brojevi a i b za koje je an1 + bn2 = 1, pa je

(aN)n1 + (bN)n2 = N. Dijeljeci aN sa n2 dobijamo aN = qn2 + r, (0 6 r 6n2 − 1), tako da prethodna jednakost postaje rn1 + yn2 = N, pri cemu je

y = bN + q. Prema tome, iznos N se moze isplatiti ako je y nenegativan cio

broj. Dakle, najveci iznos koji se ne moze isplatiti se dobija za y = −1 i jednak

je N = (n2 − 1)n1 − n2.

1.2 Osnovni stav aritmetike

Matematicari uzalud pokusavaju da otkriju pravilnosti u

nizu prostih brojeva, pa imamo razloga da vjerujemo da u

tu misteriju ljudski um nikada nece prodrijeti.

Leonard Ojler

Dokazacemo sada osnovnu teoremu aritmetike, jednostavan a znacajan re-

zultat.

Teorema 1.25 (Osnovni stav aritmetike)

Svaki cio broj a > 1 se moze na jedinstven nacin napisati u obliku proizvoda

a = pk11 p

k22 · · · pkn

n , (1.6)

pri cemu su p1 < p2 < · · · < pn prosti brojevi, dok su k1, k2, . . . , kn pozitivni

cijeli brojevi.

Dokaz

Dokazimo prvo da razlaganje (1.6) postoji. Koristimo strogi princip mate-

maticke indukcije. Tvrdnja je tacna za a = 2, jer je 2 prost broj. Pretpostavimo

da je tvrdnja tacna za sve brojeve b, za koje je 2 6 b < a. Ako je a prost, onda

je tvrdnja tacna. Ako a nije prost, onda se, na osnovu propozicije 1.20 moze

netrivijalno faktoristi. Dakle, postoje cijeli brojevi b i c, 0 < b < a, 0 < c < a,

za koje je a = bc. Kako za b i c postoji razlaganje (1.6), na osnovu induktivne

pretpostavke takvo razlaganje postoji i za a.

Dokazimo sada jedinstvenost razlaganja (1.6). Neka je

a = ql11 · ql22 · · · qlmm , (1.7)

14

Page 16: Elementarna teorija brojeva

GLAVA 1. PRVO PREDAVANJE 1.2. OSNOVNI STAV ARITMETIKE

jos jedno razlaganje broja a. Za a = 2 tvrdnja je ocigledno tacna. Pretpostavimo

da je tvrdnja tacna za sve cijele brojeve b, gdje je 2 6 b < a.

Imamo p1|ql11 · ql22 · · · qlmm , pa na osnovu propozicije 1.4 postoji i takav da

p1|qi. Kako je qi prost broj to je p1 = qi. Kracenjem sa p1 dobijamo

pk1−11 · pk2

2 · · · pknn = ql11 · · · qli−1

i · · · qlmm .

Kako je b = pk1−11 · pk2

2 · · · pknn < a razlaganja iz prethodne jednakosti su ista,

na osnovu induktivne pretpostavke. Slijedi da je n = m, pi = qi, ki = li za

1 6 i 6 n.

Jedna od vaznih tema elementarne teorije brojeva je i izucavanje svojstava

tzv. aritmetickih funkcija. To su funkcije koje preslikavaju skup prirodnih

brojeva u skup realnih ili kompleksnih brojeva. Ovdje cemo pomenuti dvije

takve funkcije.

Za svaki prirodan broj n definisemo funkcije τ(n) i σ(n) na sljedeci nacin:

1. τ(n)= broj pozitivnih djelitelja broja n,

2. σ(n)= suma pozitivnih djelitelja broja n.

Pomocu osnovnog stava aritmetike jednostavno se mogu izracunati vrijed-

nosti ovih funkcija. Neka je

n = pm11 pm2

2 · · · pmk

k , p1 < p2 < · · · < pk,

razlaganje broja n na proste faktore. Svi pozitivni djelitelji broja n imaju oblik

ps11 ps22 · · · pskk , 0 6 si 6 mi, i = 1, 2, . . . , k.

Prema tome, broj djeljitelja jednak je broju nizova (s1, s2, . . . , sk), za koje je

0 6 si 6 mi, (i = 1, 2, . . . , k).

Broj s1 moze uzeti vrijednosti 0 6 s1 6 m1, dakle, m1 + 1 razlicitih vrijed-

nosti, s2 moze uzeti m2 + 1 razlicitih vrijednosti itd. Na kraju sk moze uzeti

mk + 1 razlicitih vrijednosti. Na taj nacin dobijamo

τ(n) =

k∏i=1

(mi + 1).

Za σ(n) vrijedi

σ(n) =∑

(s1,s2,...,sk)

ps11 ps22 · · · pskk =

= (1 + p1 + · · ·+ pm11 ) · (1 + p2 + · · ·+ pm2

2 ) · · · (1 + pk + · · ·+ pmk

k ).

Zakljucujemo da formula vrijedi, jer se svaki sabirak prve sume pojavljuje tacno

jednom u razvoju drugog izraza. Vrijedi i obrnuto, razvojem izraza na desnoj

15

Page 17: Elementarna teorija brojeva

1.2. OSNOVNI STAV ARITMETIKE GLAVA 1. PRVO PREDAVANJE

strani dobijamo sabirke sume na lijevoj strani. Koristeci se formulom za sumu

clanova geometrijske progresije dobijamo

σ(n) =

k∏i=1

pmi+1i − 1

pi − 1. (1.8)

Iz prethodnih izraza direktno slijedi da su funkcije τ i σ multiplikativne, tj.

da vrijedi sljedeca propozicija.

Propozicija 1.26

Ako su brojevi m i n relativno prosti, tada vrijedi

τ(m · n) = τ(m)τ(n), σ(m · n) = σ(m)σ(n).

Propozicija 1.27

Neka su m i n prirodni brojevi i neka su

n = ps11 ps22 · · · pskk , m = pt11 p

t22 · · · ptkk ,

njihova razlaganja na proste faktore gdje si i ti mogu uzimati i vrijednost nula.

Tada je

(m,n) = pmin{s1,t1}1 p

min{s2,t2}2 · · · pmin{sk,tk}

k ,

[m,n] = pmax{s1,t1}1 p

max{s2,t2}2 · · · pmax{sk,tk}

k .

Isto tako je

m · n = [m,n] · (m,n).

Dokaz

Prve dvije tvrdnje su jasne. Isto tako, vrijedi

[m,n](m,n) = ps1+t11 ps2+t2

2 · · · psk+tkk = m · n.

Ako je p prost onda se brojevi oblika Mp = 2p − 1 nazivaju Mersenovim

brojevima. Ovi brojevi su povezani sa jos jednom vaznom klasom prirodnih

brojeva. To su savrseni brojevi. Ovi brojevi su bili posebno zanimljivi starim

Grcima. Prirodan broj nazivamo savrsenim ako je jednak sumi svojih pravih

djelitelja. To znaci da je n savrsen ako i samo ako je σ(n) = 2n. Prije dokaza

glavnog rezultata treba nam jedan pomocni.

16

Page 18: Elementarna teorija brojeva

GLAVA 1. PRVO PREDAVANJE 1.2. OSNOVNI STAV ARITMETIKE

Propozicija 1.28

Neka su a i n prirodni brojevi veci od 1 i neka je k = an − 1 prost broj. Tada

je a = 2 i n je prost broj.

Dokaz

Kako vrijedi an − 1 = (a− 1)(an−1 + an−2 + · · ·+ 1), to broj na desnoj strani

moze biti prost samo ako je a = 2. U tom slucaju, kada n ne bi bio prost, bilo bi

n = n1n2, (n1, n2 > 1), pa bismo imali 2n−1 = (2n1 −1)[2(n2−1)n1 + · · ·+ 1

],

a ovaj broj nije prost.

Ni do danas se ne zna da li savrsenih brojeva ima konacno mnogo ili ne. Isto tako

se ne zna da li postoji neparan savrsen broj. Karakterizacija parnih savrsenih

brojeva je bila poznata i starim Grcima.

Propozicija 1.29

Paran broj n je savrsen ako i samo ako postoji prost broj p takav da je Mp

prost broj i n = 2p−1(2p − 1).

Dokaz

Neka su p i Mp prosti brojevi i neka je n = 2p−1(2p − 1). Tada, prema formuli

(1.8), vrijedi

σ(n) =2p − 1

2− 1· (2

p − 1)2 − 1

2p − 2= 2p(2p − 1) = 2n,

pa je n savrsen.

Obrnuto, pretpostavimo da je n paran savrsen broj. Neka je n = 2l · k, pricemu je l > 0, a k je neparan. Na osnovu multiplikativnosti funkcije σ i formule

(1.8) zakljucujemo da vrijedi σ(n) = σ(2l)σ(k) = (2l+1 − 1)σ(k). Kako je n

savrsen, imamo σ(n) = 2n = 2l+1 · k, sto znaci da vrijedi

2l+1 · k = (2l+1 − 1)σ(k).

Kako su 2l+1 i 2l+1 − 1 relativno prosti, na osnovu propozicije 1.19, slijedi(2l+1 − 1

)|k, tj. k = (2l+1 − 1)m. Zakljucujemo da je σ(k) = 2l+1 · m. Iz

ocigledne jednakosti σ(k) = (2l+1− 1)m+m = k+m imamo da σ(k) = m+k,

pri cemu su i m i k djelitelji od k. Slijedi da je m jedini pravi djelitelj od k, pa

mora biti m = 1, pa zakljucujemo da je k prost. Dakle, broj 2l+1 − 1 je prost,

pa je prema prethodnoj propoziciji l + 1 = p prost. Zakljucujemo da vrijedi

n = 2lk = 2p−1(2p − 1).

17

Page 19: Elementarna teorija brojeva

1.3. KONGRUENCIJE GLAVA 1. PRVO PREDAVANJE

1.3 Kongruencije

Kako je moguce da matematika, koja je tvorevina

ljudskog uma, nezavisna od iskustva, bude prekrasno

prilagodena stvarnim pojavama?

Albert Ajnstajn

Djeljivost je svakako najinteresantnija osobina cijelih brojeva. To je osnova

aritmetike, odnosno, elementarne teorije brojeva.

Neka je n > 1 cio broj. Za cijele brojeve a i b kazemo da su kongruentni po

modulu n ako n|(a − b), i to oznacavamo sa a ≡ b mod n. Posljednju relaciju

nazivamo relacijom kongruentnosti po modulu n.

U sljedecoj propoziciji cemo izloziti osnovne osobine ove relacije.

Propozicija 1.30

1. Kongruentnost po modulu n je relacija ekvivalencije u skupu cijelih brojeva.

2. Brojevi a i b daju isti ostatak pri dijeljenju sa n ako i samo ako a ≡ b mod n.

3. Svaki cio broj a je kongruentan po modulu n tacno jednom broju iz skupa

{0, 1, . . . , n− 1}.

4. Ako je a ≡ a′ mod n i b ≡ b′ mod n, tada je a+ a′ ≡ b+ b′ mod n.

5. Ako je a ≡ a′ mod n i b ≡ b′ mod n, tada je aa′ ≡ bb′ mod n.

6. Ako je ac ≡ bc mod n i (c, n) = 1, tada je a ≡ b mod n.

Dokaz

1. Kako n|0, to znaci da n|a−a, pa je a ≡ a mod n, za svako a ∈ Z. Dakle, ≡je refleksivna relacija. Iz n|a−b ocigledno slijedi n|b−a, pa je ≡ simetricna.

Na kraju, neka n|a− b i n|b− c. Kako je a− c = a− b+ b− c, na osnovu

propozicije 1.2 slijedi n|a− c, pa je ≡ tranzitivna.

2. Neka je a = qn + r, b = q′n + r′, (r, r′ ∈ {0, 1, . . . , n − 1}). Tada je

a − b = (q − q′)n + r − r′. Prema tome, ako n|a − b onda na osnovu

propozicije 1.2, n|r − r′, a to je moguce samo za r = r′. Obrnuto, ako je

a = qn+ r, b = q′n+ r, r ∈ {0, 1, . . . , n− 1}, onda jasno n|a− b.

3. Slijedi direktno iz 2.

4. Ako je a ≡ a′ mod n i b ≡ b′ mod n, tada n|a − a′, n|b − b′, pa opet na

osnovu propozicije 1.2 vrijedi n|a− a′ + b− b′, tako da 4. vrijedi.

5. Iz aa′ − bb′ = aa′ − ab′ + ab′ − bb′ = a(a′ − b′) + (a − b)b′ slijedi da

n|a(a′ − b′) + (a− b)b′, tj. n|aa′ − bb′, te je i 5. tacno.

18

Page 20: Elementarna teorija brojeva

GLAVA 1. PRVO PREDAVANJE 1.3. KONGRUENCIJE

6. Iz n|(a− b)c i (n, c) = 1, na osnovu propozicije 1.19 slijedi da n|a− b.

Osobina 4. iz prethodne propozicije nam govori da je relacija kongruentnosti

kompatibilna sa sabiranjem cijelih brojeva, a osobina 5. da je kompati-

bilna sa mnozenjem. Vidjecemo znacaj tih cinjenica u sljedecem poglavlju.

Skup {k1, k2, . . . , kn} cijelih brojeva koji daju sve moguce ostatke po mod n se

naziva potpunim sistemom predstavnika po modulu n. Iz 3. se vidi da je

{0, 1, . . . , n−1} jedan takav skup. To je skup najmanjih nenegativnih ostataka.

Uocimo sada jednu osobinu binomnih koeficijenata, koja ce nam trebati za

dalja razmatranja.

Propozicija 1.31

Neka je p prost broj, a 0 < k < p. Tada p|(pk

).

Dokaz

Zaista, iz(pk

)= p(p−1)···(p−k+1)

k! , slijedi p(p− 1) · · · (p− k + 1) =(pk

)· k!. Kako

broj p dijeli broj na lijevoj strani prethodne jednakosti, p dijeli bar jedan od

faktora na desnoj strani. Kako je(pk

)jedini faktor na desnoj strani koji nije

manji od p, slijedi da p|(pk

).

Sljedeci rezultat neki nazivaju i ,,idiotskom Njutnovom binomnom formu-

lom”, dok ga neki nazivaju ,,studentskim snom”.

Propozicija 1.32

Ako su a i b cijeli brojevi i p prost broj, tada je

(a+ b)p ≡ ap + bp mod p. (1.9)

Dokaz

Na osnovu Njutnove binomne formule imamo

(a+ b)p = ap +

(p

1

)ap−1b+ · · ·+

(p

p− 1

)abp−1 + bp.

Tvrdnja slijedi na osnovu upravo dokazane cinjenice da su svi sabirci izraza

na desnoj strani prethodne jednakosti, izuzev, eventualno, ap i bp, djeljivi sa

p.

19

Page 21: Elementarna teorija brojeva

1.3. KONGRUENCIJE GLAVA 1. PRVO PREDAVANJE

Sada cemo dokazati malu Fermaovu teoremu, znacajan rezultat u elemen-

tarnoj teoriji brojeva.

Teorema 1.33

Ako je a cio, a p prost broj, tada je ap ≡ a mod p.

Dokaz

Dokazimo prvo teoremu za pozitivne cijele brojeve. Dokaz izvodimo indukcijom

po a. Za a = 1 tvrdnja je trivijalna. Pretpostavimo da je tvrdnja tacna za a > 1,

tj. da vrijedi:

ap ≡ a mod p.

Na osnovu kompatibilnosti kongruentnosti sa sabiranjem slijedi

ap + 1 ≡ a+ 1 mod p.

Na osnovu (1.9) dobijamo

(a+ 1)p ≡ ap + 1 mod p.

Tvrdnja slijedi iz posljednje dvije kongruencije, na osnovu tranzitivnosti.

Za a = 0 teorema je ocigledno tacna. Ako je a < 0, onda je −a > 0 pa

prema prethodnom vrijedi

(−a)p ≡ −a mod p.

Za p = 2 ova kongruencija postaje a2 ≡ −a mod 2, sto je ekvivalentno sa

a2 + a ≡ 0 mod 2. Kako je broj a2 + a = a(a+1) uvijek paran, tvrdnja vrijedi.

Ako je p neparan prost broj imamo −ap ≡ −a mod p, pa poslije mnozenja sa

−1 zakljucujemo da tvrdnja vrijedi.

Primjedba 1.34

Formulaciju teoreme, bez dokaza, Ferma je poslao pismom jednom prijetelju,

obecavsi da ce mu uskoro poslati i dokaz, ali to nikada nije ucinio. Sluzbeno

se smatra da je teoremu prvi dokazao Ojler, nekih stotinjak godina kasnije.

Medutim, isti takav dokaz je nasao Lajbnic, u jednom svom neobjavljenom

radu, pedesetak godina prije Ojlera.

Ovaj cemo dio zavrsiti diskusijom o tzv. linearnim kongruencijama. Neka je

n > 1 fiksiran cio broj. Ako su a i b dati cijeli brojevi, a x nepoznat cio broj,

onda se jednacina

ax ≡ b mod n (1.10)

20

Page 22: Elementarna teorija brojeva

GLAVA 1. PRVO PREDAVANJE 1.3. KONGRUENCIJE

naziva linearnom kongruencijom.

Rjesavanje ove jednacine se svodi na rjesavanje linearne Diofantove jednacine.

Zaista, ako je x0 rjesenje kongruencije (1.10), onda je ax0 + ny0 = b, za neki

y0. To znaci da je (x0, y0) rjesenje linearne Diofantove jednacine ax + ny = b.

Vrijedi i obrnuto, ako je (x0, y0) rjesenje te linearne Diofantove jednacine, onda

je x0 rjesenje kongruencije (1.10).

Propozicija 1.35

Linearna kongruencija (1.10) ima rjesenje ako i samo ako (a, n)|b. Ako je x0jedno rjesenje kongruencije (1.10), tada su

x0, x0 +n

(a, n), x0 +

2n

(a, n), . . . , x0 +

[(a, n)− 1]n

(a, n), (1.11)

sva, po modulu n medusobno nekongruentna, rjesenja te jednacine. Speci-

jalno, ako su a i n relativno prosti, tada linearna kongruencija ima jedinstveno

rjesenje.

Dokaz

Neka je d = (a, n). Da je d|b potreban i dovoljan uslov za rjesivost linearne

kongruencije slijedi iz odgovarajuceg rezultata za linearne Diofantove jednacine.

Neka je x0 bilo koje rjesenje linearne kongruencije, a (x0, y0) rjesenje li-

nearne Diofantove jednacine ax + ny = b. Za drugi dio teoreme dovoljno

je dokazati da je svako rjesenje x = x0 + tnd , (t ∈ Z), kongruentno po

modulu n jednom rjesenju iz skupa (1.11), te da su rjesenja iz (1.11) ne-

kongruentna po modulu n. Neka je t = qd + r, (0 6 r 6 d − 1). Tada

je x0 + tnd = x0 + (qd + r)nd = x0 + qn + rn

d . Medutim, jasno je da je

x0 + qn+ rnd ≡ x0 +

rnd mod n, pa je

x0 +tn

d≡ x0 +

rn

dmod n,

a posljednje je jedno od rjesenja iz (1.11).

Ako je x0 +knd = x0 +

k′nd mod n, k, k′ ∈ {0, 1, . . . , d − 1}, tada mora biti

k = k′, jer ako je k < k′, tada je (k−k′)nd ≡ 0 mod n, tj. (k−k′)n

d = sn, za neki

cio broj s, pa bismo imali k−k′

d = s, sto bi znacilo da je k−k′

d cio broj, a to je,

zbog k − k′ < d, nemoguce.

Primjer 1.36

Rijesiti jednacinu 11x ≡ 28 mod 1943.

21

Page 23: Elementarna teorija brojeva

1.3. KONGRUENCIJE GLAVA 1. PRVO PREDAVANJE

Rjesenje. Vrijedi (11, 1943) = 1, pa je rjesenje jedinstveno. Rjesavamo prvo Di-

ofantovu jednacinu 11x+1943y = 28. Pomocu Euklidovog algoritma dobijamo

530 · 11− 3 · 1943 = 1. Mnozenjem sa 28 dobijamo 14840 · 11− 84 · 1943 = 28,

pa je x = 14840 ≡ 1239 mod 1943 rjesenje nase jednacine.

Primjer 1.37

Rijesiti jednacinu 143x ≡ 4 mod 315.

Rjesenje. Ako zamijenimo 4 sa 319 onda mozemo podijeliti sa 11 i dobiti kon-

gruenciju 13x ≡ 29 mod 315. Ako zamijenimo 29 sa −286, i podijelimo sa 13

dobijamo x ≡ −22 ≡ 293 mod 315.

Latinskim kvadratom reda n se naziva kvadratna tabela sa n2 polja

na kojoj je razmjesteno n razlicitih predmeta, pri cemu svaki predmet ima

n identicnih kopija, tako da ni u jednoj vrsti i koloni nemaju dva ista pred-

meta. Kako nije bitno o kojim se predmetima radi, najcesce se uzimaju brojevi

0, 1, 2, . . . , n − 1. Za dva latinska kvadrata istog reda kazemo da su ortogo-

nalni, ako su uredeni parovi napravljeni od odgovarajucih elemenata jednog i

drugog kvadrata medusobno razliciti.

Posmatrajmo sljedeca dva latinska kvadrata treceg reda

0 1 2

2 0 1

1 2 0

0 1 2

1 2 0

2 0 1

.

Kako su svi uredeni parovi

(0, 0), (1, 1), (2, 2), (2, 1), (0, 2), (1, 0), (1, 2), (2, 0), (0, 1)

medusobno razliciti, ovi latinski kvadrati su ortogonalni.

Sa druge strane, postoje samo dva latinska kvadrata drugog reda

0 1

1 0

1 0

0 1,

koji nisu ortogonalni.

Primjedba 1.38

Izucavanje latinskih kvadrata inicirao je Ojler, postavljajuci tzv. problem 36

oficira. Naime, na jednom prijemu kod Carice u Petrogradu, na kome je bio

i Ojler, bilo je i 36 oficira. Po sest oficira je bilo iz sest razlicitih regimenti i

svi oficiri iz iste regimente bili su u razlicitim cinovima. Ojler je pitao da li

je moguce svrstati oficire u sest vrsta, sa po sest oficira, tako da ni u jednoj

22

Page 24: Elementarna teorija brojeva

GLAVA 1. PRVO PREDAVANJE 1.3. KONGRUENCIJE

vrsti i koloni ne budu dva oficira istog cina, niti dva oficira iz iste regimente.

Jezikom savremene matematike formulacija tog problema ima oblik: Postoje

li dva ortogonalna latinska kvadrata sestog reda? Mi necemo izlagati rjesenje

Ojlerovog problema jer je ono nesto komplikovanije. Odgovor na to pitanje je

negativan, jer ne postoje ortogonalni latinski kvadrati sestog reda.

Mi cemo dokazati da za svaki prost broj p > 2 postoje ortogonalni latinski

kvadrati reda p.

Neka je 0 < k < p. Formirajmo tabelu Xk = (x(k)ij )p×p na sljedeci nacin:

x(k)ij ≡ (j − 1) + (i− 1)k mod p, (i, j = 1, . . . , p).

Ako bi dva broja x(k)i,j1

i x(k)i,j2

iz i-te vrste bili jednaki, vrijedilo bi j1 − 1 +

(i− 1)k ≡ j2 − 1+ (i− 1)k mod p, sto je ekvivalentno sa j1 ≡ j2 mod p, iz cega

slijedi j1 = j2. Dakle, elementi u vrstama tabele su medusobno razliciti. Isto

tako je jednakost j − 1 + (i1 − 1)k ≡ j − 1 + (i2 − 1)k mod p, ekvivalentna sa

jednakoscu k(i1−i2) ≡ 0 mod p. Kako su k i p relativno prosti ova kongruencija

ima jedinstveno rjesenje i1 ≡ i2 mod p, pa je i1 = i2, sto znaci da su elementi

i u kolonama medusobno razliciti. Dakle, Xk je latinski kvadrat.

Propozicija 1.39

Ako je p > 2 i 0 < k < k′ < p, tada su Xk i Xk′ ortogonalni latinski kvadrati.

Dokaz

Treba dokazati da su parovi (x(k)ij , x

(k′)ij ), (i, j = 0, 1, . . . , p − 1), medusobno

razliciti. Dva takva para (x(k)i1j1

, x(k′)i1j1

) i (x(k)i2j2

, x(k′)i2j2

) su jednaki ako i samo ako

vrijedij1 − 1 + (i1 − 1)k ≡ j2 − 1 + (i2 − 1)k mod p,

j1 − 1 + (i1 − 1)k′ ≡ j2 − 1 + (i2 − 1)k′ mod p.

Na osnovu kompatibilnosti kongruentnosti sa sabiranjem, odavde slijedi

j1 − j2 + (i1 − i2)k ≡ 0 mod p,

j1 − j2 + (i1 − i2)k′ ≡ 0 mod p.

(1.12)

Oduzimanjem lijevih i desnih strana ove dvije kongruencije dobijamo

(i1 − i2)(k′ − k) ≡ 0 mod p.

Kako je 0 < k < k′ < p, to je k′ − k relativno prost sa p, pa ova kongruencija

ima jedinstveno rjesenje po modulu p, iz cega slijedi i1 = i2. Sada iz bilo koje

jednacine (1.12) lako slijedi j1 = j2.

23

Page 25: Elementarna teorija brojeva

2Drugo predavanje

Jos je Euklid dokazao da postoji bekonacnu prostih brojeva. Dokaz nam je

svima poznat. Mi cemo ovdje izloziti tri elegantna dokaza te teoreme, koji se

razlikuju od Euklidovog.

Teorema 2.1

Postoji beskonacno mnogo prostih brojeva.

Dokaz (Kummer)

Pretpostavimo suprotno, da su p1 < p2 . . . < pn svi prosti brojevi. Neka je

N = p1p2 · · · pn. Broj N − 1 > 1 ima bar jedan prosti faktor p. Slijedi da p

dijeli N − (N − 1) = 1, sto je nemoguze.

Dokaz (Goldbah)

Razliciti Fermaovi brojevi su relativno prosti. Zaista, neka je m < n i neka su

Fm = 22m

+ 1, Fm = 22n

+ 1 Fermaovi brojevi. Indukcijom se lako dokazuje

da vrijedi Fn = F0 ·F1 · · ·Fn−1 +2. Ako d|Fm i d|Fn, onda, prema prethodnoj

jednakosti d|2. Dakle, d = 1 ili d = 2. Kako su Fermaovi brojevi neparni, mora

biti d = 1. Neka je sada pn neki prosti faktor broja Fn. Tada je pn = pm, (m =n), pa postoji beskonacno mnogo prostih brojeva.

Page 26: Elementarna teorija brojeva

GLAVA 2. DRUGO PREDAVANJE

Dokaz (Saidak2005)

Neka je n > 1 bilo koji prirodan broj. Brojevi n i n+ 1 su relativno prosti, pa

oni imaju po jedan razlicit prosti faktor.Postoje, dakle, bar dva prosta broja.

Brojevi n(n+ 1) i n(n+ 1) + 1 su relativno prosti, pa se prosti faktor od

n(n+ 1) + 1 razlikuje od oba orosta faktora broje n(n+ 1). Postoje, dakle,

bar tri prosta broja. Ovaj se proces moze nastaviti beskrajno.

Medutim, pojavljivanje prostih brojeva je nepravilno i tesko predvidivo.

Izlozicemo nekoliko rezultat vezanih za ovu problematiku. Medu njima i cuvenu

Teoremu o prostim brojevima, koju je postulirao Gaus, a ciji je dokaz izuzetno

tezak.

Prvi rezultat koji navodimo je jednostavan a korisan. Za cio broj a kazemo

da je m-ti stepen, ako je a = bm, za neki cio broj b.

Teorema 2.2

Ako su a1, . . . , an u parovima relativno prosti brojevi i ako je a1 · a2 · · · an neki

m-ti stepen, tada je svaki ai neki m-ti stepen.

Dokaz

Na osnovu osnovnog stava aritmetike, neki broj x je m-ti stepen ako i samo ako

su svi eksponenti njegovih prostih faktora visekratnici od m. Prema tome, ako

je pkm neki faktor od a1 · · · an, onda p|a1 . . . an, pa p|ai, za neki i. Prema tome,

p je faktor od ai i nijednog drugog aj , (i = j), pa je pkm|ai. Zakljucujemo da

su ovog oblika svi faktori od ai, pa zakljucujemo da je ai neki m-ti stepen.

Prethodna tvrdnja nije tacna, ako ai nisu relativno prosti, za sto je jednostavno

naci primjer. Koristeci se osnovnim stavom aritmetike lako mozemo uopstiti

Pitagorin rezultat o iracionalnosti broja√2.

Teorema 2.3

Ako pozitivan cio broj m mije potpun kvadrat, onda je√m iracionalan broj.

Dokaz

Pretpostavimo suprotno, da je√m racionalan. Tada postoje relativno prosti

brojevi u i v, za koje je v2 = m ·u2. Kako su u2 i v2 relativno prosti, to v2|m, paposlije kracenja sa v2 dobijamo 1 = m1 · u2, sto bi znacilo da je u2 = 1 = m1,

a to bi znacilo da je m = v2, sto je nemoguce, jer m nije potpun kvadrat.

25

Page 27: Elementarna teorija brojeva

GLAVA 2. DRUGO PREDAVANJE

Oznacicemo sa pn n-ti prost broj. Tako je p1 = 2, p2 = 3, p3 = 5 itd. Iskori-

sticemo Euklidov dokaz za dobijanje jedne procjene o velicini prostih brojeva.

Teorema 2.4

Vrijedi: pn ≤ 22n−1

.

Dokaz

Koristimo indukciju po n. Za n = 1 tvrdnja je ocigledno tacna. Pretpostavimo

da je tvrdnja tacna za n i posmatrajmo broj p1 · · · pn + 1. Taj broj mora biti

djeljiv nekim prostim brojem p, pri cemu p ne moze biti ni jedan od brojeba

p1, p2, . . . , pn. Dakle

pn+1 ≤ p ≤ p1p2 . . . pn+1 ≤ 220

·221

· · · 22n−1

+1 ≤ 21+2+···+2n−1

+1 = 22n−1+1.

Dakle,

pn+1 ≤ 1

2· 22

n

+ 1 < 22n

.

Napomenimo da je prethodna procjena veoma gruba. Npr. za n = 4 imamo

223

= 256, dok je p4 = 7. Navescemo i jednu bolju ocjenu, koja slijedi iz tzv.

Bertranove hipoteze iz 1842. godine, jednog od najpoznatijih rezultata teorije

brojeva.

Teorema 2.5 (Bertranova hipoteza)

Za svako n ≥ 2 izmedu n i 2n postoji prost broj.

Dokaz ove teoreme izlazi izvan okvira naseg kursa. Napomenimo da je hipotezu

1852. godine dokazao cuveni ruski matematicar Cebisev.

Teorema 2.6

Vrijedi pn < 2n.

Dokaz

Koristimo indukciju po n. Za n = 2 tvrdnja vrijedi, jer je p2 = 3 < 22 = 4. Neka

tvrdnja vrijedi za n, tj. neka je pn < 2n. Na osnovu Bertranove hipoteze postoji

prost broj p za koji je 2n < p < 2n+1. Zakljucujemo da je pn+1 ≤ p < 2n+1.

26

Page 28: Elementarna teorija brojeva

GLAVA 2. DRUGO PREDAVANJE

Za realan broj x > 0 sa π(x) oznacava se broj prostih brojeva p za koje je

p ≤ x.

Godine 1793, K. F. Gaus je postulirao da se funkcija π(x) moze aproksimi-

rati funkcijom

li x =

∫ x

2

dt

ln t.

Preciznije, Gaus je postulirao sljedeci rezultat

limx→∞

π(x)

li x= 1.

Dokaz i ovog rezultata prevazilazi okvir naseg kursa. Samo cemo navesti jedan

njegov ekvivalentan iskaz:

π(x) ∼ x

lnx,

koji se naziva Teorema o prostim brojevima.

U saglasnosti sa prvim iskazom ove teoreme, treba dokazati da je

limx→∞

li xx

lnx

= 1,

sto se jednostavno dobija na osnovu Lopitalovog pravila.

Teoremu o prostim brojevima su prvi dokazali, nezavisno jedan od drugog,

Dirihle i Vale Poason. Teorema o prostom broju se moze interpretirati i jezi-

kom teorije vjerovatnoce. Klasicna vjerovatnoca p nekog dogadaja definise se

kao = p = mn , pri cemu je m broj ,,povoljnih”ishoda, a n broj svih mogucih

ishoda. Postavlja se pitanje: Ako slucajno izaberemo prirodan broj ≤ n, kolika

je vjerovatnoca da taj broj bude prost?

Ovdje je broj povoljnih mogucnosti jednak π(n), sto je,aproksimativno, jed-

nako nlnn , dok je n ukupan broj svih mogucnosti. Dakle,

p ≈ 1

lnn.

Ovo znaci da se porastom broja n prosti brojevi sve rjede pojavljuju.

Explicitna formula za π(x) ne postoji. Ko je otkrije, slava je pred njim.

Postavlja se pitanje: Kako dobiti sve proste brojeve do nekog fiksiranog broja

n. Problem je, dakle, izbaciti sve slozene brojeve od 1 do n. To je za velike n

mukotrpan posao.

Jedan od najstarijih metoda, vezana za ovaj problem, je tzv. Eratostenovo

sito. Naime, da bi se izostavili svi slozeni brojevi do n dovoljno je izostaviti one

koji imaju prosti faktor p, za koji vrijedi p <√n. Zaista, ako je a ≤ n slozen

broj, onda je a = b · c, (b, c > 1). neka je npr. b ≤ c. Ako je p prost faktor od

a tada p|bc, pa p|b ili p|c. U svakom slucaju je p2 ≤ bc = a, tj. p ≤√a. Prema

tome, da bismo dobili proste brojeve iz intervala [1, n], dovoljnu je izbaciti one

koji su djeljivi sa prostim brojem koji je ≤√n.

27

Page 29: Elementarna teorija brojeva

GLAVA 2. DRUGO PREDAVANJE

Tako dobijamo da ima 168 prostih brojeva od 1 do 1000, 135 od 1001 do

2000, 127 od 2001 do 3000 itd.

Konstatujmo da je veliki izazov za sve matematicare da se izmisli formula

koja proizvodi proste brojeve. Medu takvim formula je i poznata Ojlerova funk-

cija f(n) = n2+n+41, cije su sve vrijednosti za intervalu [0, 40] prosti brojevi.

Naravno f(41) nije prost broj.

Navescemo jos neke zanimljive osobine prostih brojeva. Oni mogu biti blizu

jedan drugog. Kako je 2 jedini paran prost broj, jedini par prostih brojeva

koji se razlikuju za 1 je (2, 3). Ostali parovi se razlikuju bar za dva, kao

(3, 5), (5, 7), (17, 19) itd. Ovakvi se parovi nazivaju blizancima. Otvoren je pro-

blem da li blizanaca ima konacno ili beskonacno mnogo. Sa druge strane, su-

sjedni prosti brojevi mogu biti veoma udaljeni jedan od drugog.

Preciznije, za svaki prirodan broj n postoji n uzastopnih prirodnih brojeva,

od kojih ni jedan nije prost. Zaista, broj (n+1)!+2 je djeljiv sa 2, broj (n+1)!+3

je djeljiv sa 3, itd broj (n+ 1)! + (n+ 1) je djeljiv sa n+ 1. Dakle, nijedan od

n uzastopnih brojeva

(n+ 1)! + 2, (n+ 1)! + 3, . . . , (n+ 1)! + n+ 1,

nije prost.

Jos jedna slavna hipoteza o prostim brojevima potice iz 1742. godine i

naziva se Goldbahova hipoteza, koja glasi: Svaki paran broj veci od 2 jednak

je zbiru dva prosta broja.

Euklidov dokaz se moze modifikovati tako da se dokaze da postoji bes-

konacno mnogo prostih brojeva odredenog oblika. Ovdje cemo dokazati jedan

takav rezultat.

Teorema 2.7

Postoji beskonacno mnogo prostih brojeva oblika 4k + 3.

Dokaz

Pretpostavimo suprotno, da su p1, p2, . . . , pn svi prosti brojevi navedeneog

oblika. Posmatrajmo broj N = 4p1p2 · · · pn − 1. Broj N je neparan i oblika

je 4k + 3. Prema tome, njegovi prosti faktori su neparni prosti brojevi, koji su

svi oblika 4k + 1 ili 4k + 3. Medutim, N ne moze imati prosti faktor oblika

4k+3, pa bi svi njegovi faktori morali biti oblika 4k+1. Ali, proizvod brojeva

tog oblika mora biti, ponovo, oblika 4k + 1. sto je kontradikcija.

Napominjemo da se, istom metodom, ne moze dokazati da ima beskonacno

mnogo prostih brojeva oblika 4k + 1. Zasto?

Zavrsicemo ovu sekciju sa jos jednim od cuvenih problema vezanih za proste

brojeve. Godine 1837 Dirihle je dokazao sljedeci rezultat

28

Page 30: Elementarna teorija brojeva

GLAVA 2. DRUGO PREDAVANJE

Teorema 2.8 (Dirihleova teorema)

Ako su a i b relativno prosti prirodni brojevi, tada u aritmetickoj progresiji

a, a+ b, a+ 2b, . . . , a+ (n− 1)b, . . .

Ima beskonacno mnogo prostih brojeva.

Sigurno najznacajniji rezultat iz teorije brojeva dokazan u 21 vijeku je sljedeci

rezultat kineskog matematicara Tao-a, iz 2007. godine:

Teorema 2.9 (Green, Tao)

Za svaki prirodan broj n postoji aritmeticka progresija sa n clanova koji su svi

prosti brojevi.

29

Page 31: Elementarna teorija brojeva

3Trece predavanje

Mozda ce mi buduca pokolenja biti

zahvalna bar zato, sto sam pokazao da

anticki matematicari nisu znali sve.

Pijer Ferma

Tri se teoreme smatraju fundamentalnim rezultatima elementarne teorije

brojeva. To su Mala Fermaova teorema, Ojlerova i Vilsonova teorema. O njima

i njihovim primjenama bice rijeci u ovom predavanju. Sve tri ove teoreme se

mogu dokazati kao jednostavne posljedice nekih osnovnih osobina prstena Zn i

osobina grupe Un jedinica tog prstena. Osim toga, Malu Fermaovu teoremu smo

dokazali u prethodnom koristeci se Nutnovom binomnom formulom i jednom

osobinom djeljivosti binomnih koeficijenata.

U kontekstu aritmetike prstena Zn to izgleda ovako

Teorema 3.1 (Ojlerova teorema)

Ako je (a, n) = 1 tada je

aφ(n) ≡ 1 mod n.

Dokaz

Element [a]n je invertibilan u prstenu Zn ako i samo ako je (a, n) = 1. In-

vertibilni elementi prstena Zn cine grupu Un, u odnosu na mnozenje. U toj

grupi ima φ(n) elemenata, pa tvrdnja slijedi iz poznate posljedice Lagranzove

teoreme.

Page 32: Elementarna teorija brojeva

GLAVA 3. TRECE PREDAVANJE

Teorema 3.2 (Mala Fermaova teorema)

Ako je p prost broj, a a cio broj, tada je

ap ≡ a mod p.

Dokaz

Ako p | a onda je tvrdnja ocigledna. Ako p - a onda, zbog φ(p) = p − 1, iz

Ojlerove teoreme slijedi ap−1 ≡ 1 mod p, pa mnozenjem ove kongruenacije sa

a doijamo tvrdnju.

Teorema 3.3 (Vilsonova teorema)

Za svaki prosti broj p vrijedi

(p− 1)! ≡ −1 mod p.

Dokaz

Znamo da je Zp polje, ako i samo ako je p prost broj. Izmnozimo sve nenulte

elemente tog polja. Dobijamo

X = [1]p · [2]p · · · [p− 1]p = [(p− 1)!]p.

Element [x] ∈ Zp je sam sebi inverzni element ako je [x2] = [1], tj. ([x]p −1)([x]p + 1 = [0]p). Kako u polju nema djelitelja nule slijedi [x]p = [1]p ili

[x] + p = [−1]p = [p − 1]p. Prema tome [1]p, [p − 1]p su jedina dva elementa

sa tom osobinom. To znaci da se, za svako [a]p razlicito od ova dva elementa,

pored [a]p u proizvodu [1]p · [2]p · · · [p − 1]p nalazi i [a]−1p pa je taj proizvod

jednak [p− 1]p = [−1]p.

Kao sto se vidi, dokazazi ovih teorema su jednostavni u okviru arit-

metike modn. Medutim ovu je aritmetiku razvio Gaus u djelu: Aritmeticka

istrazivanja, godine 1801. Teoreme o kojima govorimo su formulisane i doka-

zane mnogo ranije. Za malu fermaovu teoremu imamo sljedecu pricu. Postavio

je Ferma 1630. godine jednom svom prijatelju i napisao mu da bi mu poslao i

dokaz da nije tako dug. Ferma, medutim, taj dokaz nikada nije objavio. Prvi

dokaz je dao Ojler skoro sto godina kasnije, 1736. godine, mada je, izgleda,

dokaz dao Lajbnic, vec 1683. godine u jednom svom neobjavljenom radu.

Dacemo jos jedan dokaz Male Fermaove teoreme, razlicit od oba navedena.

Dokaz (Fermaova teorema)

Vidjeli smo da teorema ocigledno vrijedi ako p | a. Pretpostavimo da p - a.

31

Page 33: Elementarna teorija brojeva

GLAVA 3. TRECE PREDAVANJE

Posmatrajmo brojeve

a, 2a, . . . , (p− 1)a.

Ni jedan od ovih brojeva nije djeljiv sa p, jer ako p | ia povlaci p | i ili p | a, a ni

jedna od te dvije tvrdnje nije tacna. Iz istih razloga nikoja dva od prethodnih

brojeva nisu kongruentni po modulu p. Dakle, svi brojevi pri dijeljenju sa p

daju razlicite ostatke 1, 2, . . . , p− 1. Vrijedi, prema tome,

a · 2a · . . . · (p− 1)a ≡ 1 · 2 · · · (p− 1) mod p.

Imamo, prema tome,

ap−1(p− 1)! ≡ (p− 1)! mod p.

prethodna se kongruencija moze skratiti sa (p− 1)!, jer p - (p− 1)!.

Izlozicemo sada nekoliko primjena male Fermaove teoreme.

Primjer 3.4

Provjeriti da je

538 ≡ 4 mod 11.

Rjesenje. Na osnovu male Fermaove teoreme imamo 510 ≡ 1 mod 11. Dalje je

538 = (510)3 · (52)4 ≡ 13 · 34 ≡ 81 ≡ 4 mod 11.

Primjer 3.5

Odrediti one proste brojeve p za koje je 2p−1−1p potpun kvadrat.

Broj 2 nema navedenu osobinu. Kako za ostale proste brojeve p vrijedi p |2, naosnovu male Fermaove teoreme slijedi da je 2p−1−1

p cio broj. Pretpostavimo da

postoji n za koje je 2p−1−1p = n2. Tada je 2p−1−1 = pn2. Slijedi da je n neparan

broj, Kako je i p neparan, recimo, p = 2k + 1 to je (2k − 1)(2k + 1) = pn2.

Brojevi 2k−1 i 2k+1 su relativno prosti pa bar jedan od njih mora biti potpun

kvadrat.

Ako je 2k − 1 = r2 tada je 2k = r2 + 1, pa je 2p−1 = (r2 + 1)2. Slijedi da r

mora biti neparan, pa ako je r = 2s+ 1 to je 2p−1 = 4(2s2 + 2s+ 1), a ovo je

moguce samo za s = 0 i onda slijedi da je p = 3.

Ako je 2k +1 = r2 onda je 2k = r2− 1, pa je 2p−1 = (r2− 1)2. Opet r mora

biti neparan, pa ako je r = 2s+ 1 tada je 2p−1 = 16(s2 + s)2, a ovo je moguce

samo za s = 1, pa je sada p = 7.

32

Page 34: Elementarna teorija brojeva

GLAVA 3. TRECE PREDAVANJE

Primjer 3.6

Postoji beskonacno mnogo prostih brojeva oblika 4k + 1.

Rjesenje. Neka je n bilo koji prirodan broj i neka je N = (n!)2 + 1. Broj N je

neparan i veci od 1. Neka je p najmanji prosti djelitelj od N. Vrijedi p > n.

Kako je p paran on je oblika 4k+1 ili 4k+3. Kako je p djelitelj od N to vrijedi

(n!)2 ≡ −1 mod p.

Stepenovanjem sa p−12 dobijamo

(n!)p−1 ≡ (−1)p−12 mod p.

Kako p ne dijeli n! na osnovu male Fermaove teoreme vrijedi

(−1)p−12 ≡ 1 mod p.

Ako bi p bio oblika 4k + 3 vrijedilo bi

(−1)p−12 = (−1)2k+1 ≡ −1 mod p,

pa bi p|2, sto nije moguce.

Dokazali smo, dakle, da za svako n postoji prost broj p > n oblika 4k + 1,

cime je teorema dokazana.

Jedna od najvaznijih primjena male Fermaove teoreme je za utvrdivanje da

li je neki broj prost ili ne. Naime, ako se ustanovi da kongruencija an ≡ a mod n

nije tacna za neko a, onda sigurno znamo da n nije prost broj.

33

Page 35: Elementarna teorija brojeva

4Cetvrto predavanje

Primjer 4.1

Pokazati da je 117 slozen broj.

Rjesenje. Dokazimo da 2117 ≡ 2 mod 117.

Vrijedi 2117 = (27)16 · 25. Kako je 27 = 128 ≡ 11 mod 117, imamo

2117 ≡ 1116 · 25 ≡ (121)8 · 25 ≡ 48 · 25 ≡ 221 mod 17.

Dalje je 221 ≡ 113 ≡ 121 · 11 ≡ 44 mod 117. Dakle, 2117 ≡ 44 ≡ 2 mod 117.

Jos su stari Kineski matematicari postavili sljedecu hipotezu: Broj n je prost

ako i samo ako n | 2n − n. Zanimljivo je da je hipoteza tacna za n ≤ 340. Tek

1819. godine ustanovljeno je da hipoteza nije tacna za 341 = 11 · 31.Da bismo to ustanovili iskoristicemo sljedeci rezultat:

Propozicija 4.2

Ako su p i q razliciti prosti brojevi, a a cio broj, za koji vrijedi:

aq ≡ a mod p, ap ≡ a mod q.

Tada je

apq ≡ a mod pq.

Dokaz

Vrijedi apq = aq mod p. Na osnovu pretpostavke slijedi apq ≡ a mod p. Na isti

nacin dobijmo apq ≡ a mod q, iz cega slijedi tvrdnja.

Page 36: Elementarna teorija brojeva

GLAVA 4. CETVRTO PREDAVANJE

Primjer 4.3

Vrijedi 2341 ≡ 2 mod 341.

Rjesenje. Kako je 341 = 11 · 31 imamo:

211 = 25 · 25 · 25 ≡ 1 · 1 · 2 = 2 mod 31.

231 = (25)6 · 2 ≡ 106 · 2 = (100)3 · 2 ≡ 2 mod 11,

pa tvrdnja vrijedi na osnovu prethodne propozicije.

Slozen broj n nazivamo pseudo prostim ako n | 2n − 2. Iz prethodnog

primjera slijedi da je 341 pseudo prost i to je najmanji takav broj. Postoji

beskonacno mnogo pseudo prostih brojeva, sto ce slijediti iz sljedece propozicije.

Propozicija 4.4

Ako je n pseudo prost broj, tada je Mn = 2n − 1 pseudo prost.

Dokaz

Kako je n slozen broj, on se moze napisati u obliku n = rs, (1 ≤ r ≤ s < n).

Jednostavno se provjerava da vrijedi 2r − 1 | 2n − 1. Dakle, Mr |Mn, pa je Mn

slozen broj. Na osnovu pretpostavke imamo n | 2n − 2. prema tome imamo

2Mn−1 = 22n−2 = 2kn,

za neki pozitivan cio broj k. Prema tome imamo

2Mn−1 − 1 = 2kn − 1 = (2n − 1)(2n(k−1) + cdots+ 2n + 1).

Dakle Mn | 2Mn−1 − 1, tako da je

2Mn ≡ 2 modMn.

Sada cemo dati jednu primjenu Vilsonove teoreme, koja je vezana za kvadratne

kongruencije.

Propozicija 4.5

Neka je p neparan prost broj. Tada kvadratna kongruencija

x2 + 1 ≡ 0 mod p,

ima rjesenje ako i samo ako je p ≡ 1 mod 4.

35

Page 37: Elementarna teorija brojeva

GLAVA 4. CETVRTO PREDAVANJE

Dokaz

Neka je a rjesenje date kongruencije. Tada p - a. Na osnovu male Fermaove

teoreme vrijedi

1 ≡ ap−1 =(a2) p−1

2 = (−1)p−12 mod p.

Kako je p neparan prost broj to je p = 4k + 3 ili p = 4k + 1. Prvi slucaj

otpada je tada p−12 = 2k + 1 pa imamo 1 ≡ −1 mod p iz cega slijedi p|2, sto je

nemoguce. Prema tome p mora biti oblika p = 4k + 1.

Obrnuto, neka je p = 4k + 1. Vrijedi

(p−1)! = 1·2 · · · p− 1

2·p+ 1

2· · · (p−2)·(p−1) ≡ 1·(−1)·2·(−2) · · · p− 1

2·−p− 1

2mod p.

Vrijedi, dakle,

(p− 1)! ≡ (−1)p−12

(1 · 2 · · · p− 1

2

)2

mod p.

Sa druge strane, na osnovu Wilsonove teoreme je (p − 1)! ≡ −1 mod p i

cinjenice da je p−12 paran vrijedi

−1 ≡[(p− 1

2

)!

]2mod p,

pa je, dakle(p−12

)! rjesenje kongruencije.

O Ojlerovoj teoremi bice jos govora kada se budu izucavala svojstva Ojlerove

φ funkcije. Ovdje cemo dati samo jednu njenu jednostavnu primjenu.

Propozicija 4.6

Neka je (a, n) = 1. tada rekurzija ax ≡ b mod n ima rjesenje

x = aφ(n)−1b mod n.

.

Dokaz

Znamo od ranije da u ovom slucaju jednacina ima jedinstveno rjesenje. Sa druge

strane je

ax ≡ aφ(n)b ≡ b mod n.

36

Page 38: Elementarna teorija brojeva

GLAVA 4. CETVRTO PREDAVANJE

Primjer 4.7

Neka je p prost, a n bilo koji prirodan broj. Tada je

(np)!

n!pn≡ (−1)n mod p.

Rjesenje. Neka je a cio broj za koji je a ≡ 1 mod p. Tada je

a(a+ 1) · · · (a+ (p− 2) ≡ (p− 1)! ≡ −1 mod p.

Prema tome imamo

(np)!

n!pn=

(np)!

p · 2p · 3p · · ·np=

n∏r=1

[(r− 1)p+1] · [(r− 1)p+2] · · · [(r− 1)p+ p− 1].

Kako je (r − 1)p+ 1 ≡ 1 mod p, to je svaki faktor proizvoda, prema pret-

hodnom, kongruentan −1 po modulu p. Dakle,

(np)!

n!pn≡

n∏r=1

(−1) ≡ (−1)n mod p.

37

Page 39: Elementarna teorija brojeva

5Peto predavanje

5.1 Multiplikativne funkcije

Funkcije koje su definisane na skupu prirodnih brojeva, a cije su vrijed-

nosti u polju realnih ili kompleksnih brojeva nazivaju se aritmeticke funkcije.

Aritmeticku funkciju f nazivamo multiplikativnom ako iz (m,n) = 1 slijedi

f(mn) = f(m)f(n).

Vec ranije smo ustanovili da su aritmeticke funkcije τ i σ multiplikativne.

Svakako najznacajnija aritmeticka funkcija je Ojlerova φ funkcija.

Teorema 5.1

Ojlerova funkcija φ(n) je multiplikativna.

Dokaz

Neka je (m,n) = 1. Postavimo sve brojeve od 1 do mn u pravougaonu tabelu

na sljedeci nacin

1 2 3 . . . n

n+ 1 n+ 2 n+ 3 . . . 2n...

...... . . .

...

(m− 1)n+ 1 (m− 1)n+ 2 (m− 1)n+ 3 . . . mn

.

Trebamo dokazati da na ovoj tabeli ima φ(m)φ(n) brojeva koji su relativno

prosti sa mn. Napomenimo da, ako je (m,n) = 1, tada je r relativno prost sa

mn ako i samo ako je relativno prost i sa m i sa n

Page 40: Elementarna teorija brojeva

GLAVA 5. PETO PREDAVANJE 5.1. MULTIPLIKATIVNE FUNKCIJE

Neka je r pozitivan cio broj koji nije veci od n. Pretpostavimo da je (r, n) =

d > 1. Tada je svaki broj u r-toj koloni prethodne tabele djeljiv sa d, pa

nije relativno prost sa n. Sa druge strane, ako je (r, n) = 1, onda je svaki

broj iz r-te kolone relativno prost sa n. Prema tome, brojeve u tabeli, koji su

relativno prosti samn nalaze se u r-toj koloni, za koju vrijedi (r, n) = 1 i koji su

relativno prosti sa m. Takvih kolona ima φ(n). Treba jos dokazati da u svakoj

od tih φ(n) kolona ima φ(m) brojeva koji su relativno prosti sa m. Za to je

dovoljno dokazati da brojevi jedne takve kolone cine potpuni sistem ostataka

po modulum. Zaista, ako je r+k1n = q1m+r1, r+k2n = q2m+r2, pri cemu je

0 ≤ k1, k2 ≤ m−1. Ako bi bilo r1 = r2,tada bi vrijedilo (k1−k2)n = (q1−q2)m.Kako je (m,n) = 1, slijedilo bi da m dijeli k − k1, sto je nemoguce.

Konstatujmo dvije jednostavne cinjence za funkciju φ(n). Ako je p prost

broj, onda su svi brojevi 1, 2, . . . , p− 1 relativno prosti sa p pa vrijedi

φ(p) = p− 1.

Isto tako, brojevi koji nisu veci od pk i nisu relativno prosti sa pk, pri cemu je

p prost broj, moraju biti oblika s · p, (1 ≤ s ≤ pk−1) i ocigledno ih je tacno

pk−1. Vrijedi, prema tome

φ(pk) = pk − pk−1.

Kao neposrednu posljedicu prethodnih razmatranja imamo

Teorema 5.2

Ako je n = pk1pk2 · · · pkt razlaganje broja n na proste faktore, tada vrijedi

φ(n) = nt∏

k=1

(1− 1

pk

).

Primjer 5.3

Pokazati da je φ(360) = 96.

Teorema 5.4

Ako je n > 2 onda je φ(n) paran broj.

Dokaz

Ako je n = 2k, onda je φ(2k) = 2k − 2k−1 = 2k−1 i k > 1, pa je tvrdnja tacna,

u ovom slucaju.

39

Page 41: Elementarna teorija brojeva

5.1. MULTIPLIKATIVNE FUNKCIJE GLAVA 5. PETO PREDAVANJE

Ako n nije oblika 2k, onda je n djeljiv nekim neparnim prostim brojem p.

Dakle n = pkm, pri cemu je p - m. Slijedi φ(n) = φ(pk)φ(m) = pk−1(p −1)φ(m), a p− 1 je paran.

Teorema 5.5

Za svaki prirodan broj n vrijedi

n =∑d|n

φ(d).

Dokaz

Podijelimo sve brojeve iz skupa 1, 2, . . . , n u klase Cd, tako da u klasi Cd leze

oni brojevi m za koje d = (m,n). Jasno je da skupovi Cd, pri cemu d|n, cineparticiju prethodnog skupa. Medutim, broj x se nalazi u klasi Cd ako i samo

ako je (x, n) = d tj. ako i samo ako(xd ,

nd

)= 1. Prema tome, u klasi Cd ima

φ(nd

)brojeva, pa vrijedi n =

∑d|n φ

(nd

)=∑

d|n φ(d).

Definicija 5.6

Funkciju sa skupa pozitivnih cijelih brojeva u R ili C nazivamo aritmetickom.

Aritmeticku funkciju f nazivamo multiplikativnom ako vrijedi f(mn) =

f(m)f(n), kad god su m i n relativno prosti.

U prethodnoj teoremi je dokazano da je Ojlerova φ funkcija multiplikativna.

Ranije je takode pokazano da su funkcije τ i σ multiplikativne.

Teorema 5.7

Ako je f multiplikativna funkcija, tada je funkcija F (n) =∑

d|n f(d) takode

multiplikativna.

Dokaz

Neka su m i n relativno prosti. Svaki djelitelj d broja mn moze se napisati u

obliku d = d1d2, pri cemu su d1 i d2 relativno prosti i d1|m, d2|n. Prema tome

vrijedi

F (mn) =∑d|mn

f(d) =∑

d1d2|mn

f(d1d2) =

∑d1|m,d2|n

f(d1)f(d2) =∑d1|m

f(d1)∑d2|n

f(d2) = F (m)F (n).

40

Page 42: Elementarna teorija brojeva

GLAVA 5. PETO PREDAVANJE 5.1. MULTIPLIKATIVNE FUNKCIJE

Definicija 5.8

Neka su f i g aritmeticke funkcije. Tada se Dirihleov proizvod ili multiplikativna

konvolucija f ∗ g defnise na sljedeci nacin

(f ∗ g)(n) =∑

d1d2=n

f(d1)g(d2) =∑d|n

f(d)g(nd).

Jasno je da funkcije f i g simetricno ulaze u prethodnu jednakost, a to znaci

da je f ∗ g = g ∗ f. Isto tako je

f ∗ (g ∗ h)(n) =∑

d1d2d3=n

f(d1)f(d2)f(d3); (n ∈ N),

pa lako zakljucujemo da je

f ∗ (g ∗ h) = (f ∗ g) ∗ h.

Defnisimo funkciju i tako da bude i(1) = 1, i(n) = 0; (n > 1). Jednostavno se

provjerava da za svaku aritmeticku funkciju f vrijedi

f ∗ i = i ∗ f = f.

Oznacicmo sa [1] konstantnu funkciju, koja uvijek uzima vrijednost 1.

Definicija 5.9

Za aritmeticku funkciju f funkciju F = f ∗ [1] nazivamo sumacionom funkcijom

od f.

Jasno je da vrijedi: F (n) =∑

d|n f(d).

Ranije je dokazano da je F (n) = n sumaciona funkcija za Ojlerovu φ funk-

ciju.

Teorema 5.10

Dirihleov proizvod multiplikativnih funkcija je multiplikativna funkcija.

Dokaz

Neka su f i g dvije multiplikativne funkcije i (m,n) = 1. Po definiciji imamo

(f ∗ g)(mn) =∑

d1d2=mn

f(d1)g(d2). (5.1)

41

Page 43: Elementarna teorija brojeva

5.1. MULTIPLIKATIVNE FUNKCIJE GLAVA 5. PETO PREDAVANJE

Isto tako imamo

(f ∗ g)(m) =∑

i1i2=m

f(i1)g(i2), (f ∗ g)(n) =∑

j1j2=n

f(j1)g(j2).

Odavde slijedi

(f ∗ g)(m)(f ∗ g)(n) =∑

i1i2=m

f(i1)g(i2)∑

j1j2=n

f(j1)g(j2).

Kako sum i n relativno prosti to su it, (t = 1, 2) relativno prosti sa jt, (t = 1, 2),

sto implicira

(f ∗ g)(m)(f ∗ g)(n) =∑

i1j1i2j2=mn

f(i1)f(j1)g(i2)g(j2) =∑

d1d2=mn

f(d1)g(d2).

Poredeci ovu sa jednakoscu (5.1) zakljucujemo da vrijedi

(f ∗ g)(mn) = (f ∗ g)(m)(f ∗ g)(n).

Teorema 5.11

Sumaciona funkcija multiplikativne funkcije je multiplika tivna.

Dokaz

Dokaz slijedi iz prethodne teoreme i cinjenice da je [1] multiplikativna funkcija.

42

Page 44: Elementarna teorija brojeva

6Sesto predavanje

Odredicemo sada inverz od [1] u odnosu na multiplikativnu konvoluciju.

Oznaci imo ga sa µ. Zbog [1] ∗ µ = i vrijedi ([1] ∗ µ)(1) = µ(1) = 1. Isto tako

je, za n > 1, imami ([1] ∗ µ)(n) =∑

d|n µ(d). Specijalno, za prosti broj p vrijdi

([1] ∗ µ)(p) = µ(1) + µ(p) = 1 + µ(p) = 0, pa je

µ(p) = −1.

Dalje je ([1] ∗µ)(p2) = 1+µ(p)+µ(p2) = µ(p2) = 0. Indukcijom lako slijedi da

je

µ(pk) = 0, (k ≥ 2).

Ako zelimo da µ bude multiplikativna funkcija onda, za medusobno razlicite

proste brojeve p1, p2 . . . pk mora vrijediti

µ(p1p2 · · · pk) = (−1)k.

Iz navedenih osobina jasno je da funkcija µ treba biti defnisana na sljedeci

nacin.

µ(n) =

1 ako n = 1,

0 ako p2|n za neki prost brojp,

µ(p1p2 · · · pk) = (−1)k ako su p1, p2, · · · , pk medusobno razliciti prosti brojevi.

Iz prethodnih razmatranja dobijamo

Teorema 6.1

Funkcije µ i [1] su jedna drugoj inverzne, u odnosu na Dirihleov proizvod.

Page 45: Elementarna teorija brojeva

GLAVA 6. SESTO PREDAVANJE

Teorema 6.2 (Mebijusova formula inverzije)

Neka je f neka aritmeticka funkcija, a F njena sumaciona funkcija tj. F (n) =∑d|n f(d). Tada vrijedi

f(n) =∑d|n

µ(d)F(nd

).

Dokaz

Vrijedi F = [1] ∗ f , pa je µ ∗ F = µ ∗ ([1] ∗ f) = (µ ∗ [1]) ∗ f = i ∗ f = f.

Funkcija τ(n) moze biti napisana u obliku

τ(n) =∑d|n

1.

Kako je f = [1] multiplikativna funkcija na osnovu teoreme 5.11, ponovo za-

kljucujemo da je τ multiplikativna funkcija. Mebijusova formula inverzije u

ovom slucaju glasi:

1 =∑d|n

µ(d)τ(nd

).

Isto tako je

σ(n) =∑d|n

d,

a kako je i f(n) = n multiplikativna funkcija, dobijamo

n =∑d|n

µ(d)σ(nd

).

Teorema 6.3

Neka je f aritmeticka funkcija. Postoji Dirihleov inverz g funkcije f ako i samo

ako je f(1) = 0. Rekurentna formula za g data je sa

g(n)f(1) = −∑

d|n, d<n

g(d)f(nd

).

Dokaz

Ako funkciju g definisemo rekurzivno sa g(1) = 1f(1) , a za n > 1 pomocu

rekurzije u teoremi lako dobijamo da je f ∗ g = i.

44

Page 46: Elementarna teorija brojeva

GLAVA 6. SESTO PREDAVANJE6.1. PRIMITIVNI KORIJENI I INDEKSI

Izlozicemo jednu jednostavnu primjenu Mebijusove formule inverzije. Dokazali

smo da je F (n) = n sumaciona funkcija Ojlerove funkcije φ(n), tj. da vrijedi

n =∑d|n

φ(n).

Na osnovu Mebijusove formule inverzije dobijamo

φ(n) =∑d|n

µ(d)n

d.

Ova se formula moze koristiti za racunanje vrijednosti Ojlerove funkcije.

Primjer 6.4

Izracunati φ(15).

Rjesenje Djelitelji od 15 su 1, 3, 5, 15, pa imamo

φ(15) = 15µ(1) + 5µ(3) + 3µ(5) + µ(15) = 15− 5− 3 + 1 = 8.

6.1 Primitivni korijeni i indeksi

Neka su a i m pozitivni cijeli brojevi za koje je (a,m) = 1. Redom elementa

a po modulu m, u oznaci, om(a) nazivamo red elementa a u grupi jedinica Um

po modulu m. Jasno je da vrijedi

Teorema 6.5

Neka su a,m pozitivni cijeli brojevi za koje je (a,m) = 1, i ako je om(a) red

elementa a po modulu m. Tada ako an = 1 (modm), onda om(a)|n.Specijalno, om(a)|φ(m). Isto tako, ako jem = p prost broj, onda om(a)|p−1.

Primjer 6.6

o7(2) = 3.

Definicija 6.7

Broj a nazivamo primitivnim korijenom po modulu m ako je om(a) = φ(m).

Drugim rijecima, a je primitivni korijen po modulu m, ako i samo ako je grupa

Um ciklicka. Iz jednostavnog svojstva ciklickih grupa slijedi

45

Page 47: Elementarna teorija brojeva

6.1. PRIMITIVNI KORIJENI I INDEKSIGLAVA 6. SESTO PREDAVANJE

Teorema 6.8

Ako postoji primitivni korijen po modulu m onda postoji φ(φ(m)) takvih ko-

rijena.

Teorema 6.9

Neka su a,m pozitivni cijeli brojevi za koje je (a,m) = 1, i neka je om(a) red

elementa a po modulu m. Ako je ai ≡ aj mod m tada je i ≡ j mod om(a).

Teorema 6.10

Neka je om(a) = e. Tada je om(ak) = e(e,k) .

Dokaz

Ako om(ak) = l, tada je e|kl, pa e(e,k) |

lk(e,k) , pa kako su e

(e,k) i k(e,k) relativno

prosti vrijedi e(e,k) |l. Sa druge strane je

(ak)e

(e,k) = (ae)k

(e,k) ≡ 1 mod m,

tvrdnja vrijedi.

Speecijalno, om(ak) = e ako i samo ako (e, k) = 1.

46

Page 48: Elementarna teorija brojeva

7Sedmo predavanje

U sljedecoj tabeli navodimo redove elemenata po modulu 13.

broj 1 2 3 4 5 6 7 8 9 10 11 12

red 1 12 3 6 4 12 12 4 3 6 12 2

Teorema 7.1

Neka je α primitivni korijen po modulum. Tada su najmanji ostaci α, α2, . . . , αφ(m)

po modulu m neka permutacija brojeva ≤ m i relativno prosti sa m.

Teorema 7.2 (Lagranzova teorema)

Neka je f(x) =∑n

i=0 aixi, polinom stepena n ≥ 1 sa cijelim koeficijentima, a

p prost broj i p |an. Tada jednacina

f(x) ≡ 0 mod p (7.1)

ima najvise n nekongruentnih po modulu p rjesenje.

Dokaz

Indukcijom po n. Ako je n = 1 tada imamo kongruenciju

ax ≡ b mod p,

pri cemu (a, p) = 1. Znamo da ta kongruencija ima tacno jedno rjesenje. Pret-

postavimo da je n > 1. Ako jednacina (7.1) nema rjesenja onda je sve jasno.

Page 49: Elementarna teorija brojeva

GLAVA 7. SEDMO PREDAVANJE

Pretpostavimo da je α jedno njeno rjesenje, tj. da vrijedi .

f(α) ≡ 0 mod p.

Odavde slijedi da je

f(x)− f(α) ≡ 0 mod p.

Ova se jednacina svodi na jednacinu

(x− α)f1(x) ≡ modp,

pri cemu je f1(x) polinom n − 1-og stepena sa cijelim koeficijentima. Svako

rjesenje β kongruencije (7.1) koje nije kongruentno po modulu p rjesenju α

mora biti rjesenje kongruencije

f1(x) ≡ 0 mod p,

jer je (p, β − α) = 1, pa tvrdnja slijedi indukcijom.

Teorema 7.3

Neka je p prost broj, a d|p− 1. Tada kongruencija

xd − 1 ≡ 0 mod p (7.2)

ima tacno d nekongruentnih rjesenja po modulu p.

Dokaz

Napomenimo da jednacina xp−1 ≡ 1 mod p ima tacno p − 1 nekongruentnih

rjesenja po modulu p. Naime, ocigledno su 1, 2, . . . , p− 1 rjesenja te jednacine,

na osnovu male Fermaove teoreme. Sa druge strane, ako je x bilo koje rjesenje

onda p ne dijeli x pa je x kongruentno nekom od gore navedenih rjesenja. Ako je

p− 1 = kd onda je xp−1− 1 = (xd− 1) · g(x), pri cemu je g(x) polinom stepena

d(k − 1). Prema prethodnom, jednacina g(x) ≡ 0 mod p moze imati najvise

dk − d rjesenja, pa jednacina xd − 1 ≡ modp mora imati bar d rjesenja.

Kao posljedicu ove teoreme navodimo

Teorema 7.4 (Vilsonova teorema)

Ako je p prost broj onda je (p− 1)! ≡ −1 mod p.

48

Page 50: Elementarna teorija brojeva

GLAVA 7. SEDMO PREDAVANJE

Dokaz

Posmatrajmo funkciju

f(x) = (x− 1) · · · (x− p+ 1)− xp−1 + 1.

To je polinom p − 2 stepena, pa ima najvise p − 2 nekongruentnih rjesenje

po modulu p. Sa druge strane, 1, 2, . . . , p − 1 su nekongruentna rjesenja, a to

znaci da su svi koeficijenti jednaki nuli po modulu p. Uzimajuci slobodan clan

dobijamo Vilsonovu teoremu.

Teorema 7.5

Neka je p prost broj i neka d | p − 1. Tada postoji tacno φ(d) cijelih brojeva

ciji je red jednak d po modulu p.

Dokaz

Neka d | p − 1 i neka je ψ(d) broj cijelih ciji je red d. Posmatrajmo sumu∑c|d ψ(c). Neka a ima red c. Iz ac ≡ 1 mod p − 1, slijedi ad ≡ 1 mod p − 1.

Prema tome, a je rjesenje jednacine (7.2), iz cega slijedi∑

c|d ψ(c) ≤ d. Sa

druge strane, svako rjesenje jednacine (7.2) ima nek i red c, za koji vrijedi c | d,pa vrijedi i obrnuta nejednakost. Dakle, imamo

d =∑c|d

ψ(c).

Na osnovu Mebijusove formule inverzije dobijamo ψ(d) = φ(d).

Kao posljedicu dobijamo

Teorema 7.6

Ako je p prost broj, tada postoji primitivni korijen po modulu p. Prema tome,

postoji φ(p− 1) primitivnih korijena po modulu p.

Primjer 7.7

Odrediti primitivni korijen po modulu 19.

Rjesenje. Vrijedi: p−1 = 18 = 2·32. Treba, dakle, odrediti elemente reda 2 i reda

32 u grupi U19. Za prvi element mora vrijediti x2 ≡ 1 mod 19, x ≡ 1 mod 19.

Jasno je da je to broj 18. Sljedeci broj x2 mora imati red 9 tj. mora biti rjesenje

kongruencije x9 ≡ 1 mod 19. Direktnom provjerom nalazimo da su rjesenja te

jednacine data sa 1, 4, 5, 6, 7, 9, 11, 16, 17. Svako ovo rjese nje ima red 1, 3 ili 9.

49

Page 51: Elementarna teorija brojeva

GLAVA 7. SEDMO PREDAVANJE

Na taj nacin ispadaju brojevi 1, 7, 11. Tako imamo sest mogucnosti za izbor

x2. Mnozenjem ovih vrijednosti sa x1 = 18 dobijamo sljedecih sest primitivnih

korijena po modulu 19. a to su 2, 3, 10, 13, 14, 15. Lako se provjerava da je 2

primitivni korijen po modulu 9, tako da i neki slozeni brojevi imaju primitivne

korijene. Navescemo sada dva primjera slozenih brojeva, koji nemaju primitivne

korijene.

Teorema 7.8

Za k ≥ 3 broj 2k nema primitivnog korijena.

Dokaz

Pretpostavimo da je a neparan broj i da je k ≥ 3. Tada

a2k−2

≡ 1 mod 2k.

Dokaz vodimo indukcijom po k. Lako se provjerava da tvrdnja vrijedi za k = 3,

jer su brojevi 1, 3, 5, 7 rjesenja kongruencije x2 ≡ 1 mod 8. Pretpostavimo da

tvrdnja vrijedi za k. Dakle a2k−2

= 1 + b · 2k. Vrijedi; a2k−1

=(a2

k−2)2=

(1 + b · 2k)2 = 1 + 2k+1(b+ b22k−1). Tako dobijamo

a2k−1

≡ 1 mod 2k+1.

Brojevi koji su relativno porosti sa 2k su upravo neparni brojevi, pa je φ(2k) =

2k−1. Prema prethodnom, za svaki neparan broj a i svaki k ≥ 3 vrijedi

aφ(2k)

2 ≡ 1 mod 2k,

sto znaci da 2k nema primitivnog korijena.

50

Page 52: Elementarna teorija brojeva

8Osmo predavanje

Broj 1 je primitivni korijen po modulu 2, dok je 3 primitivni korijen po

modulu 4.

Teorema 8.1

Ako je m > 2 i n > 2, pri cemu je (m,n) = 1, tada broj mn nema primitivnog

korijena.

Dokaz

Neka je h = [φ(m), φ(n)], a d = (φ(m), φ(n)). Na osnovu teoreme 5.4, vrijedi

d ≥ 2. Prema tome

h =φ(m)φ(n)

d≤ φ(mn)

2.

Sa druge strane, na osnovu Ojlerove teoreme imamo aφ(m) ≡ 1 mod m. Stepe-

nujuci ovu kongruenciju sa φ(n)d dobijamo

ah =(aφ(m)

)φ(m)d ≡ 1 mod m.

Dakle, red elementa a ne moze biti φ(mn).

Moze se dokazati da vrijedi sljedeca teorema:

Page 53: Elementarna teorija brojeva

8.1. INDEKSI GLAVA 8. OSMO PREDAVANJE

Teorema 8.2

Postoji primitivni korijen broja n ako i samo ako broj n ima jedan od oblika.

n = 2, 4, pk, 2pk, (p > 2 prost broj).

Prethodni rezultat je dokazao Gaus 1801. godine.

8.1 Indeksi

Neka je α primitivni korijen po modulu p. Tada su α, α2, . . . , αp−1 nekon-

gruentni po modulu p i ni jedan nije djeljiv sa p. Prema teoremi 7.1 ti brojevi

predstavljaju neku permutaciju skupa {1, 2, . . . , p − 1}. Prema tome, za svaki

cio broj a, koji nije djeljiv sa p, postoji k za koji je a ≡ αk mod p. Ocigledno

vrijedi

Teorema 8.3

Ako je α primitivni korijen po modulu p i p - a, tada je kongruencija

a ≡ αm mod p

ekvivalentna kongruenciji

indαa ≡ m mod p− 1.

Pri tome je broj k jednoznacno odreden. Taj se broj naziva indeksom broja

a, (u odnosu na primitivni korijen α i oznacava se sa k = indα(a).

Primjer 8.4

Konstruisati tablicu indeksa po modulu 19 u odnosu na primitivni korijen 2.

Rjesenje. Trazena tablica ima oblik:

1 2 3 4 5 6 7 8 9 10 11 12 13 14 15 16 17 18

18 1 13 2 16 14 6 3 8 17 12 15 5 7 11 4 10 9.

U prvom redu su napisani brojevi, a u drugoj njihovi indeksi.

Ako je indα(a) = k, indα(b) = m, tada je indα(ab) = k+m. Ova jednakost

znaci da je multiplikativna grupa Up izomorfna aditivnoj grupi {0, 1, . . . , p−1}po modulu p, tako da indeksi imaju ososbinu koju imaju logaritmi.

Specijalno je indα(am) = m · indα(a).

52

Page 54: Elementarna teorija brojeva

GLAVA 8. OSMO PREDAVANJE 8.1. INDEKSI

Primjer 8.5

Rijesiti jednacinu 4x9 ≡ 7 mod 13.

Rjesenje. Lako se provjerava da je 2 jedan od primitivnih korijena po modulu

13. Naime, za stepena od 2 imamo

21 ≡ 2 25 ≡ 4 29 ≡ 5

22 ≡ 4 26 ≡ 12 210 ≡ 10

23 ≡ 8 27 ≡ 11 211 ≡ 7

24 ≡ 3 28 ≡ 9 212 ≡ 1

,

pri cemu su sve kongruencije po modulu 13. Postavljena jednacina eqvivalentna

je jednacini

ind24 + 9ind2x ≡ ind27 mod 12.

Iz prethodne tabele slijedi

2 + 9ind2x ≡ 11 mod 12,

tj.

9ind2x ≡ 9 mod 12,

koja je ekvivalentna sa

ind2x ≡ 1 mod 4.

Ova jednacina ima rjesenje

ind2x = 1, 5, 9.

Sa tabele citamo da su rjesenja kongruencije

x = 2, 5, 6 mod 13.

Teorema 8.6

Neka je p prost broj, a cio broj koji nije djeljiv sa p, a m pozitivan cio broj.

Neka je, dalje, d = (m, p− 1). Tada kongruencija

xm ≡ a mod p (8.1)

ili nema rjesenja ili ima tacno d rjesenja.

Dokaz

Neka je α primitivni korijen po modulu p. Neka je k = indα(a), i = indα(x).

Tada jednacina (8.1) ima oblik:

αmi ≡ αk mod p.

53

Page 55: Elementarna teorija brojeva

8.2. KVADRATNE KONGRUENCIJE GLAVA 8. OSMO PREDAVANJE

Po definiciji primitivnog korijena po modulu p ova jednacina ima rjesenje ako

i samo ako linearna kongruencija

m · i ≡ k mod p− 1

ima rjesenje.

Ako je d = (m, p− 1), onda ova jednacina ima rjesenje ako i samo ako d|ki, u tom slucaju, jednacina ima tacno d rjesenja.

Teorema 8.7

Sljedeci uslovi su ekvivalentni

1◦Jednacina (8.1) ima rjesenje.

2◦Jednacina yd ≡ a mod p ima rjesenje,

3◦Vrijedi ap−1d ≡ 1 mod p.

Dokaz

Ako je x rjesenje jednacine (8.1) tada je(x

md

)d ≡ a mod p, sto znaci da 1◦ ⇒2◦.

Neka jednacina yd ≡ a mod p ima rjesenje y. Na osnovu male Fermaove

teoreme vrijedi yp−1 ≡ 1 mod p, na osnovu cega dobijamo(yd) p−1

d ≡ 1 mod p,

tj,

ap−1d ≡ 1 mod p.

Dakle, 2◦ ⇒ 3◦.

Dokazimo, na kraju, da 3◦ ⇒ 1◦. Iz uslova 3◦, na osnovu teoreme 8.3, slijedi

kp− 1

d≡ p− 1 mod p− 1,

iz cega slijedi da d|k, pa tvrdnja vrijedi, na osnovu prethodne teoreme.

8.2 Kvadratne kongruencije

Primjenom prethodne teoreme na kvadratnu kongruenciju dobijamo

54

Page 56: Elementarna teorija brojeva

GLAVA 8. OSMO PREDAVANJE 8.2. KVADRATNE KONGRUENCIJE

Teorema 8.8 (Ojlerov kriterijum)

Neka je p neparan prost broj, a a cio broj koji nije djeljiv sa p. Tada kvadratna

kongruencija

x2 ≡ a mod p (8.2)

ima rjesenje ako i samo ako ap−12 ≡ 1 mod p.

Definicija 8.9

Cijeli brojevi a za koje jednacina (8.2) ima rjesenje, nazivaju se kvadratni

ostaci po modulu p. Ostali cijeli brojevi se nazivaju kavadratnim neostacima

po modulu p.

Kako za svaki neparan prost broj p i svaki cio broj iz skupa {1, 2, . . . , p−1}na osnovu male Fermaove teoreme vrijedi ap−1 ≡ 1 mod p, imamo(

ap−12 − 1

)(a

p−12 + 1

)≡ 0 mod p.

Odavde slijedi

ap−12 ≡ 1 mod p ili a

p−12 ≡ −1 mod p.

Prema tome, vrijedi

Teorema 8.10

Ako je p neparan prost broja, tada je cio broj a, p |a kvadratni neostatak ako

i samo ako

ap−12 ≡ −1 mod p.

Primjer 8.11

Broj 13 ima sest kvadratnih ostataka 1, 3, 4, 9, 10, 12 i sest kvadratnih neosta-

taka 2, 5, 6, 7, 8, 11.

Teorema 8.12

Neparan prost broj p ima tacno p−12 kvadratnih ostataka i isto toliko kvadratnih

neostataka.

Dokaz

Kako jednacina xp−12 ≡ 1 mod p ima najvise p−1

2 rjesenja, to znaci da ima

najvise p−12 kvadratnih ostataka. Sa druge strane, brojevi 12, 22, . . . ,

(p−12

)2su

kvadratni ostaci i, pri tome, su nekongruentni po modulu p. Zaista, ako bi dva

55

Page 57: Elementarna teorija brojeva

8.2. KVADRATNE KONGRUENCIJE GLAVA 8. OSMO PREDAVANJE

takva broja k2 i m2 bili kongruentni po modulu p imali bismo k2 ≡ m2 mod p,

Slijedi da p|k +m ili p|k −m. To je nemoguce jer su i k +m i k −m manji od

p. Prema tome, postoji nar p−12 kvadratnih ostataka.

56

Page 58: Elementarna teorija brojeva

9Deveto predavanje

Posmatrajmo kvadratnu kongruenciju

ax2 + bx+ c ≡ 0 (mod p), (p - a),

pri cemu je p neparan prost broj. Mnozenjem sa 4a dobijamo

(2ax)2 + 4abx+ 4ac ≡ 0 (mod p), (p - a),

iz cega dobijamoy2 ≡ D mod p) (9.1)

pri cemu je y = 2ax + b, D = b2 − 4ac. Prema tome, pitanje da li pocetna

kongruencija ima rjesenje ekvivalentno je pitanju: Da li je diskriminanta D

kvadratni ostatak ili ne.

Napomenimo jos da, ako je y0 rjesenje gornje kongruencije, onda je drugo

rjesenje p− y0 i ta dva rjesenja nisu kongruentni po modulu p. Zaista, imamo

(p − x0)2 = p2 − 2px0 + x20 ≡ x20 ≡ D mod p. Sa Druge strane, ako bi bilo

x0 ≡ p− x0 mod p slijedilo bi p | x0, sto nije tacno.

Da bi nasli rjesenje jednacine (9.1) treba odrediti brojeve iz skupa{1, 22, . . . ,

(p− 1

2

)2},

koji su kongruentni broju D, po modulu p.

Primjer 9.1

Rijesiti kvadratne kongruencije

3x2 − 4x+ 7 ≡ 0 mod 13.

Page 59: Elementarna teorija brojeva

GLAVA 9. DEVETO PREDAVANJE

3x2 + 7x+ 5 ≡ 0 mod 13.

Rjesenje Za prvu kongruenciju imamo x ≡ 6, 4 (mod 13). Druga kongruencija

nema rjesenja.

Koristenje Ojlerovog kriterijuma za velike p i a zahtijeva mnogo racuna.

Za njegovo skracivanje od velike koristi su tzv. Lezandrovi simboli. Neka je p

neparan prost broj, a a cio broj za koji p - a. Lezandrov simbol (a/p) =(

ap

)se definise sa

(a/p) =

{1 ako je a kvadratni ostatak po modulu p

−1 ako a nije kvadratni ostatak

U specijalnom slucaju, za p = 13 imamo

(1/13) = (3/13) = (4/13) = (9/13) = (10/13) = (12/13) = 1,

(2/13) = (5/13) = (6/13) = (7/13) = (8/13) = (11/13) = 1.

Prije svega, Ojlerov kriterijum se sada moze zapisati u obliku

U sljedecoj teoremi dajemo fundamentalne osobine Lezandrovih simbola.

Teorema 9.2

Neka je p neparan prost broj, a a, b cijeli brojevi, za koje je p - ab. Tada

1. Ako a ≡ b mod p tada (a/p) = (b/p).

2. (a2/p) = 1.

3. (a/p) ≡ ap−12 b mod p.

4. (a/p)(b/p) = (ab/p).

5. (1/p) = 1, (−1/p) = (−1)p−12 .

Dokaz

Osobine 1. i 2. su ocigledne. Osobina 3. je Ojlerov kriterijum.

(ab/p) ≡ (ab)p−12 ≡ a

p−12 · b

p−12 ≡ (a/p) · (b/p) mod p,

sto dokazuje 4.

Iz osobine (2) slijedi

1. Proizvod dva kvadratna ostatka je kvadratni ostatak. (To znaci da kva-

dratni ostaci cine grupu)

2. Proizvod dva kvadratna neostatka je kvadratni ostatak.

58

Page 60: Elementarna teorija brojeva

GLAVA 9. DEVETO PREDAVANJE

3. Proizvod kvadratnog ostatka je kvadratnog neostatka je kvadratni neosta-

tak.

Posljedica 9.3

Neka je p neparan prost broj. Tada

(−1/p) =

{1 if p ≡ 1(mod 4),

−1 if p ≡ 3(mod 4).

Dokaz

Vrijedi: (−1/p) = (−1)p−12 mod p = (−1)

p−12 .

Primjer 9.4

Izracunati (−4/41), (−9/83).

Rjesenje. Vrijedi

(−4/41) = (−1/41)(4/41) = (−1/41) = 1,

(−9/83) = (−1/83)(9/83) = (−1/83) = −1.

Primjer 9.5

Dokazati da ima beskonacno mnogo prostih brojeva oblika 4k + 1.

Dokaz. Pretpostavimo suprotno, da su p1, p2, . . . , pm svi prosti brojevi oblika

4k+1. Neka je N = (2p1p2 · · · pm)2+1. Posto je N neparan on mora biti djeljiv

nekim neparnim prostim brojem p. To znaci da je ((2p1p2 · · · pm)2 ≡ −1 mod p.

Na osnovu teoreme 9.2 vrijedi (2p1p2 · · · pm)2/p) = −1, pa bi imali (−1/p) =

−1, a to bi znacilo da p morao biri oblika 4k + 1, sto nije tacno.

Dacemo sada dokaz Gausove leme koja fundamentalna za izucavanje kva-

dratnih ostataka i neostataka. Neka je p neparan prost broj i a cio broj koji

nije djeljiv sa p. Oznacimo P = p−12 i posmatrajmo brojeve

a, 2, . . . , Pa.

Dodajuci ili oduzimajuci visekratnike od p mozemo pretpostaviti da svi ovi

brojevi leze u intervalu (−p2 ,

p2 ). To zapravo znaci da je svaki broj u gornjem

nizu kongruentan jednom od brojeva u sljedecem nizu

±1,±2, . . . ,±P.

59

Page 61: Elementarna teorija brojeva

GLAVA 9. DEVETO PREDAVANJE

Pokazimo da na ovaj nacin ne moze pojaviti isti broj k sa dva razlicita pred-

znaka, jer bi tada p||2k| ≤ p−2, a to je nemoguce. Isto tako se ne moze pojaviti

isti broj sa istim predznakom. Dakle, u posljednjem nizu se moraju pojaviti svi

brojevi iz niza ±1,±2, . . . ,±P sa oderedenim predznakom. Mnozenjem dva

niza dobijamo

a · 2a · · ·Pa ≡ (±1) · (±2) · · · (±P ) mod p.

Kracenjem sa 2, 3, . . . , P dobijamo

aP ≡ (−1)ν mod p.

Na osnovu Ojlerovog kriterijuma dobijamo

(a/p) = (−1)ν .

Teorema 9.6 (Gausova lema)

Neka je p neparan prost broj, a a cio broj koji nije djeljiv sa p i neka je ν broj

negativnih elemenata u nizu a, 2a, . . . , Pa, tada je

(a/p) = (−1)ν .

60

Page 62: Elementarna teorija brojeva

10Deseto predavanje

Primjer 10.1

Pomocu Gausove leme pokazati da je 5 kvadratni ostatak o modulu 19.

Rjesenje. Trebamo svaki od brojeva 5, 10, 15, 20, 25, 30, 35, 40, 45 dodavanjem

visekratnika od 19 smjestiti u interval (−192 ,

192 ). Dobijamo brojeve

5,−9,−4, 1, 6,−8,−3, 2, 7,

od kojih su 4 negativna.

Pomocu Gausove leme lako se moze odrediti kvadratni karakter broja 2.

Metod iz ovih razmatranja daje osnovnu ideju za dokaz Gausovog zakona uza-

jamnosti. Bice koristena sljedecjednostavna cinjenica: Ako je a realan broj, koji

nije cio, a m cio broj, tada se u interalu oblika (a, a+2m) nalazi se paran broj

cijelih brojeva. Zaista, cijeli brojevi koji se nalaze u tom intervalu su

⌊a⌋+ 1, ⌊a⌋+ 2, . . . , ⌊a⌋+ 2m,

i ima ih tacno 2m.

Neka je a = 2, a p neparan prost broj i P = p−12 . Posmatramo niz

2, 4, . . . , 2P.

U prethodnom postupku negativni ce postati samo oni koji su veci od p2 .

Treba, dakle, odrediti broj cijelih brojeva x za koje je p2 < 2x < p, odnosno,

broj cijelih brojeva x, za koje je p4 < x < p

2 . Pretpostavimo da je p = 8k + r,

pri cemu r mora pripadati skupu {1, 3, 5, 7}. Dobijamo nejednacine

2k +r

4< x < 4k +

r

2.

Page 63: Elementarna teorija brojeva

GLAVA 10. DESETO PREDAVANJE

Translacijom ulijevo za 2k, a zatim izostavljanjem brojeva iz intervala ( r2 , 2k+r2 ), kojih je paran broj, zakljucujemo da je parnost brojeva iz trazenog intervala

ista kao i parnost cijelih brojeva y za koje vrijedi

r

4< y <

r

2.

Ako je r = 1 takvih brojeva nema. Ako je r = 3 onda ima jedan, ako je r = 5

ima jedan i ako je r = 7 imaju dva. Time je dokazana sljedeca teorema:

Teorema 10.2

Broj 2 je kvadratni ostatak prostih brojeva oblika 8k±1, a kvadratni neostatak

prostih brojeva oblika 8k ± 3.

Razmatranja za broj 3 su slicna. U ovom slucaju posmatramo brojeve

3, 6, . . . , 3P. Svi su ovi brojevi manji od 3p2 Oni koji ce poslije pomjeranja

postati negativni nalaze se u intervalu (p2 , p). Trazimo, dakle, cijele brojeve x

za koje jep

6< x <

p

3.

napisimo p u obliku p = 12k + r, pri cemu r moze uzeti vrijednosti 1, 5, 7 i 11.

Nejednakost dobija oblik

2k +r

6< x < 4k +

r

3,

i kao malocas svodi se na nejednacinu

r

6< x <

r

3.

Rjesenja ne postoje u slucajevima kada je r = 1, jedno rjesenje ako je r = 5 ili

r = 7, a dva rjesenja ako je r = 11. Tako vrijedi

Teorema 10.3

Broj 3 je kvadratni ostatak za proste brojeve oblika 12k ± 1, a kvadratni neo-

statak za proste brojeve oblika 12k ± 5.

Iz prethodna dva rezultata zakljucujemo sljedece. Kvadratni karakter broja 2

ne zavisi od prostog broja p nego samo od ostatka r koji nastaje njegovim

dijeljenjem sa 8. I pri tome brojevi sa ostatkom r i 8 − r imaju isti karakter.

U drugom rezultatu je pokazano da kvadratni karakter broja 3 zavisi samo od

ostatka r′ pri dijeljenju sa 12 i da ostaci r′ i 12− r′ imaju isti karakter.

Ojler je postulirao tu tvrdnju za proizvoljan slucaj, ali je nije znao dokazati.

Ta Ojlerova tvrdnja je ekvivalentna cuvenom Gausovom zakonu o reciprocitetu

i moze se jednostavno dokazati pomocu Gausove leme.

62

Page 64: Elementarna teorija brojeva

GLAVA 10. DESETO PREDAVANJE

Teorema 10.4 (Ojlerova teorema)

Neka je p neparan prost broj, a a cio broj takav da p - a. Tada kvadratni

karakter broja a po modulu p ne zavisi od broja p, nego samo od ostatka r

pri dijeljenju broja p sa 4a. Pored toga, brojevi r i 4a− r imaju isti kvadratni

karakter.

Dokaz

Posmatramo brojeve a, 2a, . . . , Pa i pitamo se: Koji ce od njih, poslije pomje-

ranja u interval −p2 < x < p

2 biti negativni.

To su ocigledno visekratnici broja a koji pripadaju intervalima(i · p− p

2, i · p

), (i = 1, 2, . . . , b),

pri cemu je b najveci broj za koji je b ≤ a2 . Dijeljenjem sa a problem se svodi

na prebrojavanje cijelih brojeva iz intervala(i · pa− p

2a, i · p

a

), (i = 1, 2, . . . , b).

Ako napisemo p u obliku p = 4ka+r, (0 ≤ r < 4a), pomjeranjem za 4ki ulijevo

i izostavljanjem parnog broja clanova iz intervala(−2k +

(i− 1

2

), i · r

a

)dobijamo intervale ((

i− 1

2

)r

a, ir

a

), (i = 1, 2, . . . , b).

Prethodni intervali zavise samo od r, a ne od pojedinih prostih brojeva. Time

je prvi dio teoreme dokazan.

Ako umjesto r stavimo 4a− r onda i-ti interval postaje((i− 1

2

)4a− r

a, i4a− r

a

),

tj. (4i− 2−

(i− 1

2

) ra, 4i− i

r

a

).

Izostavljanjem parnih brojeva dobijamo(−(i− 1

2

) ra,−i r

a

),

a ovaj je interval, bar sto se tice parnosti brojeva u njemu ekvivalentan pola-

znom.

63

Page 65: Elementarna teorija brojeva

GLAVA 10. DESETO PREDAVANJE

Teorema 10.5 (Gausov princip reciprocnosti)

Neka su p i q razliciti neparni prosti brojevi. Tada je(p/q)·(q/p)= (−1)

p−12 · q−1

2 .

Dokaz

Pretpostavimo da je p ≡ q mod 4. Mozemo pretpostaviti da je p > q, te da je

p = 4a + q. Pri tome p - a i q - a. Dalje je jasno da p i q daju isti ostatak pri

dijeljenu sa 4a. To, na osnovu Ojlerove teoreme, znaci da vrijedi(a/p)=(a/q).

Sa druge strane je(p/q)=((4a+ q)/q

)=(4a/q

)=(a/q).

Analogno je (q/p)=((−4a+ p)/p

)=(− 4a/p

)=(− 1/p

)(a/p).

Prema tome imamo (p/q)·(q/p)=(− 1/p

),

a ovo znaci da je (p/q)·(q/p)= 1,

ako su i p i q oblika 4k + 1, a(p/q)·(q/p)= −1,

ako su i p i q oblika 4k + 3.

U slucaju p ≡ q mod 4 imamo p + q ≡ 0 mod 4. To znaci da p i q daju

suprotne ostatke po modulu 4. Imamo(p/q)=((4a− q)/q

)=(4a/q

)=(a/q).

Na isti nacin (q/p)=(a/p).

Kako su, na osnovu Ojlerove teoreme, ponovo(a/p)i(a/q)jednaki, teorema

je dokazana.

64

Page 66: Elementarna teorija brojeva

11Jedanaesto predavanje

Ako su p i q neparni prosti brojevi i bar jedan oblika 4k+1, tada je p−12

q−12

paran broj. Taj je broj neparan samo u slucaju kada su i p i q oblika 4k + 3.

Tako se Gausov princip reciprocnosti moze formulisati u sljedecem obliku.

Posljedica 11.1

1. Ako su p i q prosti brojevi, od kojih je bar jedan oblika 4k + 1 tada je

(a/p) = (b/p).

2. Ako su oba p i q oblika 4k + 3 tada je

(a/p) = −(b/p).

Izlozicemo jos jedan dokaz ove vazne teoreme, koji je smislio Ajzenstajn.

Dokaz (Drugi dokaz zakona reciprocnosti)

Posmatrajmo pravougaonik u xy ravni, ciji su vrhovi:

(0, 0), (0,q

2), (

p

2, 0), (

p

2,q

2).

Neka je R unutrasnjost pravougaonika. U unutrasnnjosti ocigledno ima p−12

q−12

tacaka sa cjelobrojnim koordinatama. Ideja dokaza je da se ove tacke prebroje

na dva razlicita nacina.

Jednacina dijagonale D (0, 0), (p2 ,q2 ) glasi y = q

px, (0 ≤ x ≤ p2 ). Jasno je da

se ni jedna tacka ne nalazi na dijagonali D.

Page 67: Elementarna teorija brojeva

GLAVA 11. JEDANAESTO PREDAVANJE

Neka je T1 dio unutrasnjosti ispod dijagonale, a T2 dio iznad dijagonale. U

intervalu (0, kqp ) imamo ⌊kqp ⌋ cjelobrojnih tacaka. To znaci da na vertikalnoj

liniji (k, 0), (k, kqp ), (1 ≤ k ≤ p−12 ) postoji tacno ⌊kq

p ⌋ cjelobrojnih tacaka.

Prema tome postoji∑ p−1

2

k=1

⌊kqp

⌋cjelobrojnih tacaka u T1. Analogno, u dijelu

T2 ima∑ q−1

2

k=1

⌊kpq

⌋tacka. Tako dobijamo

p− 1

2· q − 1

2=

p−12∑

k=1

⌊kq

p

⌋+

q−12∑

k=1

⌊kp

q

⌋. (11.1)

Neka je a proizvoljan neparan cio broj takv da p - a. Posmatrajmo skup

{a, 2, a, . . . , Pa}, pri cemu je P = p−12 . Dokazimo da vrijedi jednakost

(a/p) = (−1)∑P

k=1⌊ kap ⌋. (11.2)

Dijeljenjem svakog visekratnika ka sa p dobijmo

ak = qkp+ rk, (k = 1, 2, . . . , P ).

Pri tome je, za svaki k, 1 ≤ rk ≤ p − 1. Slijedi kap = qk + rk

p , iz cega slijedi

qr =⌊kap

⌋. Prema tome imamo

ak =

⌊ka

p

⌋· p+ rk. (11.3)

Broj ν iz Gausove leme jednak je broju onih rk, za koje je p2 < rk < p.

Oznacicemo ovakve rk sa sk, a ostale sa tk. Prema tome, p− s1, p− s2, . . . , p−sν , tν+1, . . . , tP cine permutaciju skupa 1, 2, . . . , P. Zbog toga vrijedi

P∑k=1

k =ν∑

k=1

(p− sk) +P∑

k=ν+1

tk,

tj.P∑

k=1

k = ν · p−ν∑

k=1

sk +

P∑k=ν+1

tk. (11.4)

Sumirajuci po k u relaciji (11.3) dobijamo

P∑k=1

ak = p

P∑k=1

⌊ka

p

⌋+

P∑k=1

rk. (11.5)

Oduzimajuci (11.4) od (11.5) dobijmo

(a− 1)

P∑k=1

k = p

(P∑

k=1

⌊ka

p

⌋− ν

)+ 2

ν∑k=1

sk. (11.6)

66

Page 68: Elementarna teorija brojeva

GLAVA 11. JEDANAESTO PREDAVANJE

Vodeci racuna da je a ≡ p ≡ 1 mod 2 dobijamo

ν ≡P∑

k=1

⌊ka

p

⌋mod 2,

a ovo znaci da vrijedi (11.2).

Ako su p i q razliciti neparni prosti brojevi, na osnovu prethodnog dobijamo

(p/q)(q/p) = (−1)∑ q−1

2k=1 ⌊ kp

q ⌋+∑ p−1

2k=1 ⌊ kq

p ⌋.

Gausov zakon reciprocnosti dobijamo poredenjem posljednje jednakosti sa

(11.1).

Primjer 11.2

Izracunati (34/97).

Rjesenje. Prije svega je 34 = 2 · 17. Broj 97 je prost oblika 8k + 1. Zbog toga

je (2/97) = 1, pa je (34/97) = (17/97). Kako su 17 i 97 prosi i nisu oba

oblika 4k + 3 to je, na osnovu principa reciprociteta (17/97) = (97/17). Kako

je 97 ≡ 12 mod 17 To je

(34/97) = (12/17) = (3/17) = (17/3).

Posto je 17 ≡ −1 mod 3, na kraju dobijamo (34/97) = (−1/3) = −1.

67

Page 69: Elementarna teorija brojeva

12Dvanaesto predavanje

12.1 Neprekidni razlomci

Kontinuantom niza a0, a1, . . . , an realnih brojeva nazivamo determinantu

Cn = [a0, a1, . . . , an] =

∣∣∣∣∣∣∣∣∣∣∣∣∣∣

a0 1 0 0 0

−1 a1 1 0 0

0 −1 a3 0 0...

......

. . ....

...

0 0 0 an−1 1

0 0 0 −1 an

∣∣∣∣∣∣∣∣∣∣∣∣∣∣.

Vrijedi

C0 = a0, C1 = a0a1 + 1, c2 = a0 + a2 + a0a1a2, . . .

Razvijanjem po elementima posljednje vrste dobijamo rekurentnu formulu

Cn = Cn−1an + Cn−2, (n > 1). (12.1)

Razvojem po prvoj vrsti dobijamo

Cn = a0[a1, . . . , an] + [a2, . . . , an], (n > 1). (12.2)

Ako je a0 = a1 = . . . = an = 1, onda obje formule daju rekuryiju za Fi-

bonacijeve brojeve, tako da, u tom slucaju imamo Cn = Fn.

Page 70: Elementarna teorija brojeva

GLAVA 12. DVANAESTO PREDAVANJE12.1. NEPREKIDNI RAZLOMCI

Teorema 12.1 (Ojlerova teorema)

Kontinuanta Cn dobija se kada se proizvodu a0 · a1 · · · an dodaju svi izrazi,

koji se iz njega dobiju izostavljanjem jednoh ili vise faktora oblika aiai+1, (i =

0, . . . , n− 1).

Dokaz

Vrijedi C0 = a0, C1 = a0a1+1, pa je tvrdnja tacna za n = 0 i n = 1. Pretposta-

vimo da tvrdnja vrijedi za k < n i neka je n > 1. Svi sabirci koji iz proizvoda

a0 · · · an izostavljanem faktora an−1 · an sastoje se od proizvoda a0 · · · an−2 i

svih proizvoda koji iz njega nastaju izostavljanjem po dva uzastopna faktora

i ti sabirci jasno cine kontinuantu niza a0, . . . , an−2. U svim ostalim sabircima

kao faktor se mora pojaviti an, (jer se on ne pojavljuje jedino u sabircima u

kojima je izostavljen par an−1an), a njihov zbir je [a0, . . . , an−1] · an, pa prva

tvrdnja vrijedi na osnovu formule (12.1).

Teorema 12.2

Vrijedi:[a0, a1, . . . , an]

[a1, a2, . . . , an]= a0 +

1

a1 +1

a2+1

...+ 1an

. (12.3)

Dokaz

Formula (12.2) moze se napisati u obliku.

[a0, . . . , an]

[a1, . . . , an]= a0 +

1[a1,...,an][a2,...,an]

.

Primjenom ove formule na nazivnik desne strane itd., dobijamo tvrdnju.

Izraz na desnoj strani jednakosti (12.3) naziva se neprekidni ili verizni raz-

lomak.

Vrijedi i obrat prethodne teoreme

Teorema 12.3

Svaki se pozitivan racionalan broj na jedinstven nacin moze prikazati kao ne-

prekidni razlomak.

69

Page 71: Elementarna teorija brojeva

12.1. NEPREKIDNI RAZLOMCIGLAVA 12. DVANAESTO PREDAVANJE

Dokaz

Neka ab racionalan broj, ciji su brojnik i nazivnik relativno prosti. Primjenom

Euklidovog algoritma na brojeve a i b dobijamo

a = q0b+ r0,

b = q1r0 + r1,

r0 = q2r1 + r2,...

rn−2 = qnrn−1 + 1.

Odavde lako dobijamo jednakost

a

b= q0 +

1

q1 +1

q2+1

...+ 1qn

.

Ako je a0, a1, . . . beskonacan niz onda se konacni neprekidni razlomci nazi-

vaju konvergentama tog niza. Konvergentu na desnoj strani jednakosti (12.3)

nazivacemo n-tom konvergentom niza i oznacavati sa An.

Sljedeca teorema se naziva Osnovna teorema o neprekidnim razlomcima, a

uspostavlja vezu izmedu ,,susjednih ”konvergenti:

An−1 = a0 +1

a1 +1

a2+1

...+ 1an−1

i An = a0 +1

a1 +1

a2+1

...+ 1an

.

Teorema 12.4

Oznacimo Cn = [a0, a1, . . . , an], Dn = [a1, a2, . . . , an]. Vrijedi

CnDn−1 − Cn−1Dn = (−1)n−1.

Drugim rijecima vrijedi

An −An−1 =(−1)n−1

Dn−1Dn. (12.4)

Isto tako je

An −An−2 =(−1)nanDn−2Dn

. (12.5)

Dokaz

Na osnovu (12.1) vrijedi

CnDn−1 − Cn−1Dn = (anCn−1 + Cn−2)Dn−1 − Cn−1(anDn−1 +Dn−2) =

70

Page 72: Elementarna teorija brojeva

GLAVA 12. DVANAESTO PREDAVANJE12.1. NEPREKIDNI RAZLOMCI

−(Cn−1Dn−2 − Cn−2Dn−1).

Ponavljajuci ovaj postupak dobijamo prvu tvrdnju teoreme. Preostale dvije

jednakosti se jednostavno dobijaju iz dokazane jednakosti.

Jednostavna posljedica ove teoreme je

Posljedica 12.5

Kontinuante Cn i Dn su relativno prosti brojevi.

Primjer 12.6

Rijesiti Diofantovu jednacinu

ax+ by = c, (12.6)

pri cemu su a, b, c cijeli brojevi.

Rjesenje Ako je d = (a, b) tada jednacina (12.6) ima rjesenje ako i samo ako

d | c. Ako jednacina ima rjesenje x0 i ako je x bilo koje rjesenje, tada je

a

d(x− x0) = − b

d(y − y0).

Kako su ad i b

d relativno prosti odavde dobijamo x−x0 | bd , tj. vrijedi x = x0+t

bd ,

za neki cio broj t. Sada iz prethodne jednakosti slijedi y = y0 − tad , tako da

opste rjesenje jednacine ima oblik

x = x0 + t · bd, y = y0 − t · a

d, (t ∈ Z).

Da bi odredili rjesenje (x0, y0) mozemo postupiti na sljedeci nacin: Posma-

trajmo prvo specijalan slucaj c = 1. Ako je

a

b=

[q0, q1, . . . , qn]

[q1, q2, . . . , qn]=Cn

Dn.

Na osnovu prethodne posljedice vrijedi a = Cn, b = Dn. Sa druge strane, na

osnovu teoreme 12.4 vrijedi

CnDn−1 − Cn−1Dn = (−1)n−1,

pa dobijamoaDn−1 − bCn−1 = (−1)n−1.

Na taj nacin, ako je n neparan broj, tada je x0 = Dn−1, y0 = −Cn−1, a ako je

n paran broj, onda je x0 = −Dn−1, y0 = Cn−1.

Na kraju, ako su (x0, y0) rjesenja jednacine ax+ by = 1, tada su (cx0, cy0)

rjesenje polazne jednacine.

71

Page 73: Elementarna teorija brojeva

12.1. NEPREKIDNI RAZLOMCIGLAVA 12. DVANAESTO PREDAVANJE

Teorema 12.7

Neka je An n-ta konvergenta niza a0, a1, . . . . Tada je

A2i < A2i+2, A2i+3 < A2i+1, A2i < A2j+1, (i, j ≥ 0).

Dokaz

Da parne konvergente cine monotono rastuci, a neparne konvergente monotono

opadajuci niz sljijedi iz jednakosti (12.5). Iz jednakosti (12.4) slijedi da je A2i <

A2i+1. Ako su i < j proizvoljni onda je A2i < A2j jer parne konvergente cine

monotono rastuci niz , i, zbog prethodne nejednakosti, A2j < A2j+1.

Prema prethodnoj teoremi parne konvergente cine monotono rastuci niz

ogranicen s gornje strane, a neparne konvergente cine monotono opadajuci

niz ogranicen sa donje strane. Prema tome oba ta niza konvergiraju. Sa

druge strane za svaku kontinuantu Cn se indukcijom lako dokazuje da vri-

jedi Cn > n. To znaci da da kada u (12.4) pustimo da n → ∞ dobijamo

An − An−1 → 0, (n → ∞), a to znaci da i parne i neparne konvergente kon-

vergiraju jedinstvenom broju α

Dokazimo da je taj broj iracionalan. Pretpostavimo suprotno. Tada za svako

n vrijedi A2n < α < A2n+1. Prema tome imamo 0 < α− A2n < A2n+1 − A2n.

Sa druge strane, na osnovu (12.4) vrijedi A2n+1 − A2n = 1D2nD2n+1

. Prema

tome, ako je α = ab , imamo

0 <a

b− C2n

D2n<

1

D2nD2n+1,

iz cega slijedi

0 < aD2n − bC2n <b

D2n+1.

Kako je D2n+1 > 2n + 1 to je za dovoljno veliko n ispunjeno bD2n+1

< 1, pa

bi se cio broj aD2n − bC2n nalazio u intervalu (0, 1), sto je nemoguce. Tako je

dokazana

Teorema 12.8

Konvergente beskonacnog niza a0, a1, . . . konvergiraju nekom iracionalnom

broju α.

72

Page 74: Elementarna teorija brojeva

GLAVA 12. DVANAESTO PREDAVANJE12.1. NEPREKIDNI RAZLOMCI

Za broj α iz prethodne teoreme kaze se da je predstavljen beskonacnim nepre-

kidnim razlomkom

α = An = a0 +1

a1 +1

a2+1

...an−1+ 1an+ 1

...

.

73

Page 75: Elementarna teorija brojeva

13Trinaesto predavanje

Najjednostavniji primjer za ilustraciju prethodne teoreme je niz sastavljen

od samih jedinica. Postavlja se pitanje kojem iracionalnom broju konvergiraju

kontinuante tog niza.

Theorem 13.1

Konvergente niza sastavljenog od jedinica konvergiraju Zlatnom presjeku.

Dokaz

Dokaz sljiedi iz poznate cinjenice da je Cn = Fn, pri cemu je Fn Fibonacijev

broj. Prema tome

limn→∞

An = limn→∞

Fn

Fn−1=

√5 + 1

2,

na osnovu Bineove formule.

Primjer 13.1

Odrediti iracionalni broj odreden nizom 3, 6, 1, 4, 1, 4, . . . .

Rjesenje. Neka je y iracionalan broj odreden periodicnim dijelom 1, 4, 1, 4, . . . .

Tada je y odreden i nizom 1, 4, y. Drugim rijecima vrijedi

y = 1 +1

4 + 1y

= 1 +y

4y + 1=

5y + 1

4y + 1.

Page 76: Elementarna teorija brojeva

GLAVA 13. TRINAESTO PREDAVANJE

Odavde dobijamo

y =1 +

√2

2.

Ako je α broj koji trazimo, taj je broj odreden nizom 3, 6, y iz cega slijedi

α = 3 +1

6 + 11+

√2

2

=14−

√2

4.

Dokazimo sada da se svaki iracionalan broj moze predstaviti kao beskonacan

neprekidan razlomak. Neka je α iracionalan. Formiracemo dva niza brojeva:

jedan cjelobrojni (a0, a1, . . .) takav da su svi a1, a2, . . . pozitivni i jedan niz

iracionalnih brojeva x0, x1, . . . na sljedeci nacin:

Uzecemo x0 = α, a0 = ⌊x0⌋. Broj x0 − a0 je iracionalan i 0 < x0 − a0 < 1.

Zbog toga je x1 = 1x0−a0

> 1 takode iracionalan. Uzmimo sada a1 = ⌊x1⌋.Sigurno je a1 ≥ 1 i 0 < x1−a1 < 1 je iracionalan broj, pa se prethodni postupak

moze ponovljati neograniceno. Na taj se nacin mogu formirati trazeni nizovi

po sljedecem pravilu

xn =1

xn−1 − an−1, an = ⌊xn⌋, (n = 0, 1, . . .).

Odavde dobijamo

xn−1 = an−1 +1

xn, (n = 1, 2, . . .),

odnosno

x0 = a0 +1

x1= a0 +

1

a1 +1x2

= · · · = a0 +1

a1 +1

a2+1

...an−1+ 1an+ 1

xn+1

.

Koristeci teoremu 12.2 dobijamo

x0 =C ′

n+1

D′n+1

,

gdje je

C ′n+1 = (a0, a1, . . . , an, xn+1), D

′n+1 = (a1, a2, . . . , an, xn+1).

Koristeci formulu (12.1) slijedi

x0 =Cnxn+1 + Cn−1

Dnxn+1 +Dn−1. (13.1)

Pri tome su Cn, Dn, (n = 1, 2, . . .) konvergente niza a0, a1, a2, . . . . Imamo,

dakle,

x0 −Cn

Dn=Cnxn+1 + Cn−1

Dnxn+1 +Dn−1− Cn

Dn=

(−1)n

(Dnxn+1 +Dn−1)Dn.

75

Page 77: Elementarna teorija brojeva

GLAVA 13. TRINAESTO PREDAVANJE

Dalje je xn+1 > an+1, sto implicira

| x0 −Cn

Dn|< 1

(Dnan+1 +Dn−1)Dn=

1

Dn+1Dn<

1

(n+ 1)n.

Pustajuci da n → ∞ dobijamo α = x0 = limn→∞Cn

Dn, cime je teorema doka-

zana. U dokazu prethodne teoreme dokazana je i sljedeca

Theorem 13.2

Ako su Cn

Dnkonvergente iracionalnog broja α, onda je

|α− Cn

Dn| < 1

Dn+1Dn≤ 1

D2n

.

Primjer 13.2

Prikazati zlatni presjek Φ =√5+12 u obliku neprekidnog razlomka.

Rjesenje. U ovom slucaju je a0 = ⌊Φ⌋ = 1. Dalje je

x1 =1

Φ− ⌊Φ⌋=

1√5−12

= Φ,

pa je a1 = 1. Produzavajuci proces dobijamo ϕ = (1; 1, 1, . . .).

Primjer 13.3

Broj√23 predstaviti kao neprekidan razlomak.

Rjesenje. Postupajuci u skladu sa prethodnom teoremom dobijamo

x0 =√23 = 4 + (

√23− 4) a0 = 4

x1 = 1x0−⌊x0⌋ = 1√

23−4= 1 +

√23−37 a1 = 1

x2 = 1x1−⌊x1⌋ = 7√

23−3= 3 +

√23−32 a2 = 3

x3 = 1x2−⌊x2⌋ = 2√

23−3= 1 +

√23−37 a3 = 1

x4 = 1x3−⌊x3⌋ = 7√

23−4= 8 + (

√23− 4) a4 = 8.

Vidimo dalje da je x5 = x1, iz cga slijedi da je x6 = x2 itd. Na taj nacin

dobijamo niz 4, 1, 3, 1, 8, 1, 1, 8, . . . .

Primjer 13.4

Odrediti konvergente broj π.

76

Page 78: Elementarna teorija brojeva

GLAVA 13. TRINAESTO PREDAVANJE

Rjesenje. Vrijedi π ≈ 3.141592653.... Imamo

x0 = π = 3 + (π − 3) a0 = 3

x1 = 1x0−⌊x0⌋ = 1

0.141592653... = 7.06551330... a1 = 7

x2 = 1x1−⌊x1⌋ = 1

7.06551330... = 15.99659440 . . . a2 = 15

x3 = 1x2−⌊x2⌋ = 1

0.06551330... = 1.00341723 . . . a3 = 1

x4 = 1x3−⌊x3⌋ = 1

0.00341723... = 292.63467 . . . a4 = 292.

Inace, za broj π nije poznata reprezenrtancija u obliku neprekidnog razlomka,

koja bi na bilo koji nacin bila pravilna. Sa druge strane, vec je Ojler nasao

takve reprezentacije vezane za broj e. Navescemo ovdje sljedece

e−1e+1 = (0; 2, 6, 10, 18, . . .) aritmeticki nize2−1e2+1 = (0; 1, 3, 5, 7, 9 . . .) aritmeticki niz

e = (2; 1, 2, 1, 1, 4, 1, 1, 8, 1 . . .) parni brojevi razdvajaju dvije jedinice

Iz posljednjeg prikaza slijedi da je broj e iracionalan. Dacemo ovdje jos

jedan, jednostavan, dokaz te vazne cinjenice.

Theorem 13.3

Dokazati da je broj e iracionalan.

Dokaz

Koristimo se poznatom formulom

e =∞∑i=0

1

i!.

Pretpostavimo da je e = ab racionalan broj i uzmimo prirodan broj n > b. Tada

je

N = n!

(a

b−

n∑i=0

1

i!

)prirodan broj. Sa druge strane je

N = n! ·∞∑

i=n+1

1

i!=

∞∑i=n+1

n!

i!.

Odavde slijedi

N =1

n+ 1+

1

(n+ 1)(n+ 2)+ · · ·+ 1

(n+ 1)(n+ 2) · · · (n+ k)+ · · · .

Slijedi

N <1

n+ 1+

1

(n+ 1)(n+ 2)+

1

(n+ 2)(n+ 3)+

1

(n+ 3)(n+ 4)+ · · · .

77

Page 79: Elementarna teorija brojeva

GLAVA 13. TRINAESTO PREDAVANJE

Zakljucujemo da je

N <1

n+ 1+

(1

n+ 1− 1

n+ 2

)+

(1

n+ 2− 1

n+ 3

)+ · · · .

Slijedi 1 < N < 2n+1 , sto je nemoguce, jer je N prirodan broj.

78

Page 80: Elementarna teorija brojeva

14Cetrnaesto predavanje

14.1 Pitagorine trojke

Rjesavacemo Diofantovu jednacinu

x2 + y2 = z2. (14.1)

Njena se rjesenja nazivaju Pitagorinim trojkama. Ako je (x, y, z) Pitagorina

trojka, onda su i sve trojke (±x,±y,±z) takode Pitagorina. U daljem cemo naci

pozitivna rjesenja ove jednacine. Najpoznatija Pitagorina trojka je, naravno,

(3, 4, 5). Druga, malo manje poznate su npr. (5, 12, 13) i (12, 35, 37).

Jos je sam Pitagora otkrio da postoji besjonacno mnogo trojki, ali su, tek

u Euklidovim elementima, pronadene sve trojke.

Neka je (x, y, z) rjesenje. Pretpostavicemo da ne postoji cio broj koji dijeli

sve x, y, z, jer je jasno da je za svaki cio broj a i trojka (ax, ay, az), takode

rjesenje. Prvo zakljucujemo da su svaka od dva broja x, y i z medusobno rela-

tivno prosti. To znaci da su x i y razlicite parnosti. Ne mogu oba ta broja

biti neparni. Zaista, ako je x = 2n + 1, y = 2m + 1, tada je x2 + y2 =

4(n2 + m2 + m + n) + 2, pa slijedi da je z2 paran broj. Ali, tada z mora

biti paran broj, pa je z2 mora biti djeljiv sa 4. To je nemoguce, jer broj

4(n2 +m2 +m+ n) + 2 ocigledno nije djeljiv sa 4.

Prema tome brojevi x i y su jedan paran, a jedan neparan. Uzmimo da je

x paran a y neparan. Tada je i z neparan pa su

1

2(z − y),

1

2(z + y)

Page 81: Elementarna teorija brojeva

14.1. PITAGORINE TROJKE GLAVA 14. CETRNAESTO PREDAVANJE

cijeli brojevi, pa je i njihov proizvod cio broj. Ti su brojevi i relativno prosti,

jer ako bi neki broj dijelio oba, onda bi dijelio i njihov zbir i razliku, pa bi taj

broj dijelio i y i z, a to je nemoguce, jer su ta dva broja relativno prosti.

Vrijedi, medutim

1

2(z − y) · 1

2(z + y) =

(x

2

)2

.

Na osnovu osnovnog teorema aritmetike odavde zakljucujemo da su oba broja12 (z − y) i 1

2 (z + y) potpuni kvadrati, pa ako je z − y = 2a2, z + y = 2b2. Tada

je x2 = 4a2b2. Zakljucujemo da pozitivna rjesenje jednacine ima oblik

x = 2ab, y = b2 − a2, z = a2 + b2, (14.2)

pri cemu su a i b pozitivni i b > a. Brojevi a i b, ocigledno, moraju biti relativno

prosti. Osim toga, uvijek jedan paran, a jedan neparan.

Obrnuto, ako su x, y, z dati posljednjim formulama (14.2), pri cemu su a i b

relativno prosti i razlicite parnosti, onda se lako provjerava da je x2 + y2 = z2.

Dokazimo jos da je trojka data sa (14.2) primitivna. Kako su a i b razlicite

parnosti, to je z neparan. Prema tome, ako neki prosti broj p dijeli x, y i z,

onda je p > 2. Prema tome, p | (z + y) i p | (z − y), pa p | 2a2 i p | 2b2, iz cega

slijedi p | a i p | b, sto je nemoguce.

Teorema 14.1

Sva rjesenja jednacine x2 + y2 = z2 data su sa

y = 2abt, y = t(b2 − a2), z = t(a2 + b2),

pri cemu su a, b, t proizvoljni cijeli brojevi, za koje je (a, b) = 1.

Primjer 14.2

Dokazati da je poluprecnik upisane kruznice u Pitagorin trougao sa cjelobroj

nim stranicama uvijek vio broj.

Rjesenje. Uporedivanjem povrsina lako dobijamo jednakost xy = r(x+ y + z),

pri cemu su x i y katete, z hipotenuza, a r poluprecnik upisane kruznice. Na

osnovu formula (14.2) lako se vidi da je

r =xy

x+ y + z,

cio broj.

80

Page 82: Elementarna teorija brojeva

GLAVA 14. CETRNAESTO PREDAVANJE14.2. JEDNACINA X4 + Y 4 = Z2.

14.2 Jednacina x4 + y4 = z2.

Teorema 14.3 (Fermaova teorema)

Diofantova jednacina x4 + y4 = z2 nema pozitivnih cjelobrojnih rjesenja.

Dokaz

Neka je (a, b, c) rjesenje jednacine a4 + b4 = c2. Neka je (a, b) = d. Tada je

a = αd, b = βd, za neke cijele, relativno proste α i β. Slijedi (αd)4+(βd)4 = c2.

Ako uzmemo γ = d2c, tada je, ocigledno, (α, β, γ) takode rjesenje jednacine.

Mozemo, dakle, odmah pretpostaviti da su a i b relativno prosti. Kako je c > 0

mozemo pretpostaviti da je c najmanje moguce.

Ocigledno je (a2, b2, c) Pitagorina trojka. Medutim kako je (a, b) = 1 to je i

(a2, b2) = 1, pa je to, u stvari primitivna Pitagorina trojka. Prema prethodnom,

postoje relativno prosti pozitivni cijeli brojevi m,n razlicite parnosti za koje je

a2 = 2mn, b2 = m2 − n2, c = m2 + n2.

Ako bi m bio paran, a n neparan onda bi imali b2 ≡ 3 (mod 4). Kako je, sa

druge strane, b neparan to mora biti b2 ≡ 1 (mod 4). Dobijena kontradikcija

pokazuje da je m neparan, a n paran broj. Neka je n = 2q. Tada je a2 = 4mq,

pa je(a2

)2= mq. Kako su m i n relativno prosti, to su i m i q relativno

prosti, pa iz prethodne jednakosti slijedi da su i m i q potpuni kvadrati. Neka

je m = t2, q = u2.

Sa druge strane je n2 + b2 = m2, a kako su m in relativno prosti to je

(n, b,m) primitivna Pitagorina trojka. Zbog toga postoje pozitivni, relativno

prosti, cijeli brojevi v i w za koje je

n = 2vw, b = v2 − w2, m = v2 + w2.

Iz prve jednakosti slijedin

2= vw = q = u2,

pa kako su v i w relativno prosti, to su i v i w potpuni kvadrati. Stavimo

v = r2, w = s2. Sada jednacina v2 + w2 = m postaje v4 + w4 = t2. Prema

tome (v, w, t) je rjesenje nase jednacine.

Sa druge strane imamo

0 < t ≤ t2 = m ≤ m2 < m2 + n2 = c,

sto je suprotno pretpostavci da je c najmanje moguce. Time je teorema doka-

zana.

81

Page 83: Elementarna teorija brojeva

15Petnaesto predavanje

15.1 Pelova jednacina

Vidjecemo, sada, kako se rjesava tzv. Pelova jednacina. Vjerovatno prvi put

u matematici ta se jednacina javila u tzv. Arhimedovom problemu sa stokom.

Naime, On je postavio problem da se odredi broj pojedinih vrsta od osam vrsta

stoke ciji su brojevi vezani sa sedam linearnih odnosa i dva odnosa koji govore

da se neki brojevi potpuni kvadrati. Poslije jednostavnog racuna problem se

svodi na jednacinu

x2 − 4729494y2 = 1.

Pokazano je (1880) da najmanje rjesenje problema predstavlja broj sa 41 cifrom.

Oko 650. godine nove ere indijski matematicar Brahmagupta je napisao:

Osoba koja za godinu dana rijesi jednacinu x2 − 92y2 = 1 je matematicar.

Njegova tvrdnja ima smisla, jer je najmanje rjesenje jednacine x = 1151, y =

120 i nije ga lako naci numericki. Diofantova jednacina oblika

x2 −Ny2 = 1,

pri cemu je N dati cio broj, naziva se Pell-ova jednacina. To je jos jedno od

pogresnih imena u Istoriji matematike. Naime, nije utvrdeno da matematicar

Pell ima mnogo veze sa ovom jednacinom. Pogresno ime ja dao Ojler, koji je

imao mnogo veze sa ovom jednacinom.

Slucaj N = 0 je trivijalan. Rjesenja su tada x = ±1, dok y moze biti

proizvoljan. Ako je N = −1 tada jednacina postaje x2 + y2 = 1 pa su jedina

rjesenje x = ±1, y = 0 i x = 0, y = ±1. Ako je N ≤ −2, tada je, opet, rjesenje

x = ±1, y = 0.

Page 84: Elementarna teorija brojeva

GLAVA 15. PETNAESTO PREDAVANJE 15.1. PELOVA JEDNACINA

Prema tome, mozemo se ograniciti na slucaj N > 0. Ako je N potpun

kvadrat npr. N = m2 onda je, opet, rjesenje jednostavno, jer tada jednacina

postaje x2 − (my)2 = 1, pa su joj rjesenja x − my = 1, x + my = 1, ili

x−my = −1, x+my = −1.

Prema tome, mozemo preci na najinteresantniji slucaj, kada je N pozitivan

i nema kvadratnih faktora. Smatra se da je Ferma prvi iznio tvrdenje da ova

jednacina ima beskonacno mnogo rjesenja. Prvi potpuni dokaz dao je Lagranz.

Napomenimo sljedece cinjenice o rjesenjima

1. Ako je (x, y) netrivijalno rjesenje, tada su (−x, y), (x,−y), (−x,−y) takoderjesenja. Dakle, rjesenja se javljaju u cetvorkama, pa se uvijek mozemo

ograniciri na pozitivna rjesenja.

2. Kako je 1 + Ny2 = x2 netrivijalno rjesenje bi se moglo naci racunajuci

vrijednost izraza 1 +Ny2 za razlicite vrijednosti y i ustanoviti kada je taj

izraz potpun kvadrat. Metod je trivijalalan ali je, naravno, nepraktican.

Tako je 1 + 92 · 1202 = 11512, tako da je x = 1151, y = 120 rjesenje

jednacine x2 − 92y2 = 1.

3. Ako su (a, b) rjesenja onda je a2 −Nb2 = (a− b√N)(a+ b

√N) = 1. Zbog

toga iracionalni brojevi oblika a + b√N igraju znacajnu ulogu u trazenju

rjesenja. Iz konvencionalnih razloga reci cemo da broj a + b√N proizvodi

rjesenje.

Lako se vidi da, ako r i s proizvode rjesenje tada to cini i rs. Naime, ako je

r = a+b√N, s = c+d

√N, tada je rs = (ac+bdN)+(ad+bc)

√N. Provjerava

da rs takode proizvodi rjesenje. Zaista, ako je x = ac+ bdN, y = ad+ bc, tada

je

x2 −Ny2 = a2c2 + 2acbdN + b2d2N2 − a2d2N − 2abcdN − b2c2N =

= c2(a2 − nb2) + d2N(−a2 + b2N) = c2 − d2N = 1.

Specijalno, svaki pozitivan stepen od r takode proizvodi rjesenje.

Sada cemo dokazati da, ako Pelova jednacina ima rjesenje onda postoji i

najmanje rjesenje. Pretpostavimo da r = a + b√N i s = c + d

√N proizvode

rjesenja, pri cemu su a, b, c, d > 0. Tada je r < s ako i samo ako a < c.

Ako a > c tada je a2 > c2. Kako je a2 = 1+Nb2, a c2 = 1+Nd2 dobijamo

da je b ≥ d, a to znaci da je r ≥ s.

Obrnuto, ako je a < c, tada je a2 < c2, pa je b2 < d2 tj. b < d pa je r < s.

Prema tome, ako a + b√N proizvodi rjesenje, onda je najmanje rjesenje ono

koje se doboje za najmanje a. Pokazacemo sada kako se pomocu najmanjeg

rjesenja moze naci beskonacno mnogo rjesenja, drugih rjesenja.

Ako je α2 −Nβ2 = 1 tada je jasno (α2 −Nβ2)n = 1 = (α+√Nβ)n · (α+√

Nβ)n, za svaki cio broj n. Prema tome, ako uzmemo x, y tako da bude

x+√Ny = (α+

√Nβ)n, x−

√Ny = (α−

√Nβ)n,

83

Page 85: Elementarna teorija brojeva

15.1. PELOVA JEDNACINA GLAVA 15. PETNAESTO PREDAVANJE

onda x+√Ny ocigledno proizvodi rjesenje.

Rjesavanjem ovog sistema po x i y dobijamo beskonacno mnogo rjesenja.

Jedan od metoda za dobijanje najmanjeg rjesenja je i sljedeci metod koji

se osniva na neprekidnim razlomcima. za to cemo iskoristiti sljedecu cinjenicu

o kvadratnim iracionalnostima, koju necemo dokazivati.

Ako je n prirodan broj, bez kvadratnih faktora tada, uz oznaku a0 = ⌊√N⌋,

vrijedi: √N + a0 = (2a0; a1, a2, . . . , an−1, an).

Teorema 15.1

Ako je N prirodan broj, bez kvadratnih faktora, tada postoji konvergenta An =Cn

Dnbroja

√N takva da su x = Cn, y = Dn rjesenja Pell ove jednacine x2 −

Ny2 = 1.

Dokaz

Neka su Cn−1

Dn−1i Cn

Dnuzastopne konvergente od

√N, koje neposrerdno prethode

prvom 2a0.

Prema formuli (13.1) vrijedi

√N =

Cnxn+1 + Cn−1

Dnxn+1 +Dn−1,

pri cemu je xn+1 =√N + a0. Uvrstavanjem ove vrijednosti u prethodnu jed-

nakost dobijamo

NDn + (a0Dn +Dn−1)√N = a0Cn + Cn−1 + Cn

√N.

Zbog toga sto je√N iracionalan a sve su ostalo cijeli brojevi odavde dobijamo

dvije jednakosti:

a0Cn + Cn−1 = DnN, a0Dn +Dn−1 = Cn.

Na osnovu teoreme 12.4 imamo

CnDn−1 − Cn−1Dn = (−1)n−1,

iz cega slijedi

Cn(Cn − a0Dn)− (a0Cn −DnN)Dn = (−1)n−1.

C2n −ND2

n = (−1)n−1.

Ako je[broj n neparan onda je teorema dokazana. Ako je n paran onda se isti

metod primjeni na konvergente ispred kraja drugog perioda tada ce clan anbiti u stvari a2n+1.

84

Page 86: Elementarna teorija brojeva

GLAVA 15. PETNAESTO PREDAVANJE 15.1. PELOVA JEDNACINA

Ilustrujmo ovu teoremu sa dva primjera

Primjer 15.2

Rijesiti jednacinu x2 − 21y2 = 1.

Rjesenje. Vrijedi√21 = 4, 1, 1, 2, 1, 1, 8. Tu je n = 5. Konvergente su

41 ,

51 ,

92 ,

235 ,

327 ,

5512 . Rjesenje su, znaci, x = 55, y = 12.

Primjer 15.3

Rijesiti jednacinu x2 − 29y2 = 1.

Rjesenje Vrijedi√29 = 5, 2, 1, 1, 2, 10. Ovdje je n = 4. Konvergente su

5

1,11

2,16

3,27

5,70

13.

To znaci da je x = 70, y = 13 rjesenje jednacine x2 − 29y2 = −1. Da bi nasli

rjesenje nase jednacine odredujemo sljedecih pet konvergenti

727

135,1524

283,2251

418,3775

701,9801

1820.

Rjesenje je, dakle, x = 9801, y = 1820.

85